PDA

نسخه کامل مشاهده نسخه کامل : ◄◄ اتــاق تــجــزیــه و تــرکــیــبــات ►►



صفحه ها : 1 [2] 3

m_honarmand_j
22-05-2007, 20:09
سوال چهارم روز دوم مرحله ی دوم المپیاد کامپیوتر امسال : n شکارچی خرس در کشوری با 1386 شهر مستقر اند . از هر شهری به هر شهر دیگر راه است (همبند است). طبق مقررات این گروه در هیچ زمانی نباید دو نفر از این گروه در یک شهر باشند . هنگامی که این افراد در مجموعه ای از شهر ها مستقر اند و بخواهند به مجموعه ای دیگر از شهر ها (مسلما باید تعداد شهر های برابر تعداد افراد گروه باشد)بروند به نوبت حرکت می کنند و در هر نوبت تنها یک نفر می تواند به یکی از شهر هایی که مجاور شهر کنونی اش است برود که در آن کسی دیگر نباشد .( در انتها مهم نیست که چه کسی در کدام شهر است)
آیا این افراد می توانند از هر مجموعه شهر اولیه با هر نوع جاده بین شهر ها به هر مجموعه شهر انتخابی بروند . اگر نمی توانند در چه حالتی و اگر می توانند حدلقل پس از چند روز می توان مطوئن بود که به شهر های جدید می رسند؟
جواب : اگر تعداد افراد از 1386 کمتر مساوی باشد همیشه می توانند این کار را بکنند .بدترین حالت هنگامی رخ می دهد که نقشه ی شهر ها به صورت که مسیر باشد یعنی جاده ها را بتوان پشت سر هم چید و هر شهر به دو شهر بعدی و قبلی ( در صورت وجود) راه داشته باشد و همه ی افراد گروه در n شهر اولیه ی مسیر باشند و بخواهند به n شهر پایانی مسیر بروند که در این حالت تعداد روز های لازم برابر است با :
(1386-n)*n چون هر نفر مجبور است 1386-n روز در راه باشد .

m_honarmand_j
22-05-2007, 20:20
سلام
یک جدول با دو سطر و n ستون داریم . در خانه های این جدول دست کم دو به توان n مهره وجود دارد . در هر حرکت می توان از یک خانه که بیش از 1 مهره در آن باشد دو مهره برداشت و یک مهره در خانه ی بالایی یا سمت راستی اش قرار دهیم . ثابت کنید میتوان مهره هارا طوری جابجا کرد که یک مهره به خانه ی راست-بالا برسد . (مرحله ی دوم بیست و سومین المپیاد ریاضی ایران )

سلام
خونه ی آخر رو 1 در نظر بگیرید . خونه های قبلی و پایینی شو 2/1 در نظر بگیرید و به همین ترتیب هر خونه ای و که با m حر کت می شه از اون به خونه ی آخر رسید و 1 تقسیم بر دو به توان m در نظر بگیرید . در نتیجه خونه ی اول 1 بر روی دو به توان n می شه . که در این حالت شرط لازم و کافی برای اونکه بتوان یک مهره رو به خونه ی آخر برسونیم اینه که در ابتدا مجموع ارزش های مهره ها بیشتر مساوی 1 باشد که این چنین است . این یک ایده ی جالب ه که یک سوال دیگر که با این ایده حل می شه رو می زارم .
البته می توان این مسئله رو با استقرا هم حل کرد.

m_honarmand_j
22-05-2007, 20:27
اول برات آرزوی موفقیت در المپیاد می کنم
دوم فکر نمی کردم نمونه سوال المپیاد کامپیوتر اینجوری باشه
اگه ممکنه چند تا دیگه هم بذار لطفا

اولا ممنون و دوما کلا سوالات المپیاد کامپیوتر باهالا . من نفهمیدم منظور شما از نمونه سوال چی هست . اگه منظورتون سوالات مرحله ی دوم ه ، ماله امسال و کامل گزاشتم و اگر غیر از اینه لطفا دقیق تر بگید .

m_honarmand_j
22-05-2007, 20:35
bego kodomaro hal nakarD vasat hallesh konam.:5:

خیلی خوشحال شدم که بلا خره یکی پیدا شد که مسئله حل کنه .
و اگر آقای kaakaa شما هم از این به بعد فارسی تایپ کنید ممنون می شم.

kaakaa
23-05-2007, 02:17
ok
خب پس بذار یه سوال هم من بذارم:
یک n ضلعی منتظم داریم که روی یکی از راس هاش عدد 1 و روی بقیه راس ها عدد 0 نوشته شده است. در هر مرحله یک m ضلعی منتظم که راس های ان روی راس های n ضلعی ابتدایی است انتخاب می کنیم (بدیهی است که باید n بر m بخشپذیر باشدو توجه شود که m عددی ثابت نیست و برای هر m با شرایط گفته شده مجاز به انجام عملیا ت هستیم)و به اعداد روی راس های این m ضلعی انتخاب شده 1 واحد می افزاییم.حال با انجام این عملیات ایا می توانیم تمام اعداد روی راس های n ضلعی را به عددی برابر تبدیل کنیم؟!!!:5:

m_honarmand_j
23-05-2007, 02:29
[ برای مشاهده لینک ، لطفا با نام کاربری خود وارد شوید یا ثبت نام کنید ]
13) یک جوجه در گوشه ی پایین و سمت چپ مستطیلی به طول 99 و عرض 59 قرار گرفته است. این جوجه می خواهد با تعدادی «حرکت»، خود را به گوشه ی بالا سمت راست این مستطیل برساند. جوجه در هر حرکت می تواند به اندازه ی توانی از دو (یعنی 1 یا 2 یا 4 یا ... واحد) به سمت راست یا بالا حرکت کند، به شرط آن که از جدول خارج نشو د. او به چند طریق می تواند به مقصد برسد به طوری که کم ترین تعداد «حرکت» را انجام داده باشد؟

الف) 5760 ب) 126 ج) 2880 د) 362880 ﻫ) هیچ کدام

جواب : طول را به مبنای دو ببرید و تعداد یک هایش را بشمارید . مثلا a تا . همین کار را برای عرض انجام دهید . مثلا b تا . جواب برابر انتخاب b از a+b است که برابر است با فکر کنم 126 .

m_honarmand_j
23-05-2007, 02:35
ok
خب پس بذار یه سوال هم من بذارم:
یک n ضلعی منتظم داریم که روی یکی از راس هاش عدد 1 و روی بقیه راس ها عدد 0 نوشته شده است. در هر مرحله یک m ضلعی منتظم که راس های ان روی راس های n ضلعی ابتدایی است انتخاب می کنیم (بدیهی است که باید n بر m بخشپذیر باشدو توجه شود که m عددی ثابت نیست و برای هر m با شرایط گفته شده مجاز به انجام عملیا ت هستیم)و به اعداد روی راس های این m ضلعی انتخاب شده 1 واحد می افزاییم.حال با انجام این عملیات ایا می توانیم تمام اعداد روی راس های n ضلعی را به عددی برابر تبدیل کنیم؟!!!:5:


سلام
خیر . فرض کن که این اعداد بردار هستند . در اول یک بردار از مرکز n ضلعی به جایی که 1 هست کشیده شده است . در هر انتخاب مجموع بردار هایی که اضافه می شوند برابر 0 است و هیچ گاه اندازه ی بردار تغییر نمی کند و این در حالی ایت که در آخر مجموع بردار ها برداری صفر باید بشود .

m_honarmand_j
23-05-2007, 02:39
[ برای مشاهده لینک ، لطفا با نام کاربری خود وارد شوید یا ثبت نام کنید ]
14) مورچه ای می خواهد در مکعب زیر از رأس A، با حرکت روی پاره خط ها،‌به رأس B (رأس مقابل A) برود.

فرض کنید در هر حرکت مورچه از یک سر پاره خط به سر دیگر آن می رود. می دانیم بعد از 5 حرکت مورچه روی رأس B قرار دارد. او به چند طریق می توانسته این مسیر را طی کرده باشد؟

الف) 36 ب) 48 ج) 60 د) 90 ﻫ) 120

جواب : گزینه ی ج یعنی 60 . با حالت بندی که دو حرکت اضافی رو به بالا و پایین بره یا غیره جواب رو میده .

m_honarmand_j
23-05-2007, 02:42
[ برای مشاهده لینک ، لطفا با نام کاربری خود وارد شوید یا ثبت نام کنید ]
15) در یک ردیف 33 خانه قرار دارد، که با اعداد 1 تا 33 شماره گذاری شده اند. زِبل خان در یک طرف این خانه ها قرار دارد (در کنار خانه ی شماره ی 33). در هر صبح یک توپ در یکی از خانه ها قرار می دهیم. زبل خان هر روز ظهر اگر توپی دید به آن شلیک می کند و توپ می ترکد (حداکثر در هر روز به یک توپ شلیک می کند و فقط همان توپ می ترکد). اگر یک توپ در خانه ی i باشد، زبل خان خانه های پشت آن را نمی بیند (خانه های 1 تا i-1). دود حاصل از ترکیدن یک توپ در خانه ی i ام باعث می شود که زبل خان تا بعد از ظهر روز بعد نیز خانه های پشت آن را نبیند و در نتیجه اگر روز بعد توپی در آن خانه ها گذاشته شود،‌زبل خان آن را نمی بیند و به آن تیراندازی نمی کند. ما 33 روز وقت داریم تا 33 توپ در این خانه ها بگذاریم و همچنین در هر خانه حدّاکثر می توانیم یک بار توپ قرار دهیم. در پایان 33 روز، به اندازه ی مجموع شماره ی خانه هایی که توپ در آن ها قرار دارد به ما جایزه می دهند. بیشترین جایزه ای را که می توان به دست آورد چقدر است؟

الف) 136 ب) 240 ج) 256 د) 272 ﻫ) 289

جواب : گزینه ی د . به ترتیب از طرف زبل خان توپ ها را می چینیم و در آخر تعداد توپ های باقی مانده همه در طرف دیگر که زبل خان اونجا نیست قرار می گیرند .

m_honarmand_j
23-05-2007, 02:44
[ برای مشاهده لینک ، لطفا با نام کاربری خود وارد شوید یا ثبت نام کنید ]
16) چند جدول از اعداد 0 تا 8 داریم که هر دو خانه ی مجاور (دارای یک ضلع مشترک) در آن دقیقا یکی از دو خاصیت زیر را داشته باشند:

* باقی مانده ی تقسیم اعداد آن دو خانه بر 3 برابر باشد

* خارج قسمت تقسیم اعداد آن دو خانه بر 3 برابر باشد

یکی از این جدول ها در شکل نشان داده شده است.

الف) 6 ب) 18 ج) 36 د) 72 ﻫ) 144

جواب : گزینه ی د . (نمی دونم چرا)

m_honarmand_j
23-05-2007, 02:46
[ برای مشاهده لینک ، لطفا با نام کاربری خود وارد شوید یا ثبت نام کنید ]
17) بازی «تنبل کش» یک بازی یک نفره است که روی یک جدول انجام می شود. در ابتدای بازی اعداد 1 تا 8 به ترتیب نامعینی در 8 تا از خانه های جدول قرار گرفته اند و یکی از خانه های جدول خالی است. در هر حرکت می توان عدد یکی از خانه های مجاور ضلعی ِ خانه ی خالی را به خانه ی خالی انتقال داد. هدف بازی این است که با حداقل تعداد انتقال،‌اعداد جدول به صورت شکل زیر، مرتّب شوند.

آقای «تنبل»، قصد دارد بدون انجام دادن بازی، حدس بزند که حداقل تعداد انتقال های لازم برای مرتب کردن یک جدول چند حرکت است. از این رو، وی برای هر یک از اعداد 1 تا 8،‌تعداد حرکت های لازم برای انتقال آن عدد به مکان مطلوب در جدول نهایی (با فرض خالی بودن تمام خانه های دیگر) را محاسبه کرده و مجموع این اعداد (K) را به عنوان حدس خود در نظر می گیرد.

اگر حداقل تعداد انتقال های لازم برای مرتب کردن جدول و رسیدن به جدول نهایی را A بنامیم، کدام یک از گزینه های زیر درست است؟ فرض کنید جدول های ابتدایی مورد بحث، همواره پس از متناهی حرکت مرتب می شوند.

الف) برای تمامی جدول های اولیه

ب) برای تمامی جدول های اولیه

ج) برای تمامی جدول های اولیه و جدولی اولیه وجود دارد که

د) برای تمامی جدول های اولیه

ﻫ) هیچ کدام


جواب : گزینه ی ج ( نمی دونم چرا)

m_honarmand_j
23-05-2007, 02:47
[ برای مشاهده لینک ، لطفا با نام کاربری خود وارد شوید یا ثبت نام کنید ]

18) در بازی خرگوش کشی یک جدول 1385*1385 داریم که در هر خانه ی آن یک خرگوش قرار دارد. «مردِ چکش زن» بازی را شروع می کند. او در ابتدا یک خرگوش را به دلخواه خود با چکش می کشد می کشد. سپس در هر مرحله، اگر در سطر یا آن ستون را بکشد (به دلخواه یکی از آنها بکشد). در غیر این صورت، خرگوش زنده ای را از هر کجای جدول به دلخواه می کشد و بازی ادامه پیدا می کند. هدف ما پیدا کردن تعداد روش هایی است که مرد چکش زن می تواند همه ی خرگوش ها را بکشد. باقیمانده ی این عدد بر 23 چند است؟


الف) 0 ب) 1 ج) 2 د) 21 ﻫ) 22

جواب : گزینه ی الف (نمی دونم چرا)

m_honarmand_j
23-05-2007, 02:49
[ برای مشاهده لینک ، لطفا با نام کاربری خود وارد شوید یا ثبت نام کنید ]
19) در بازی خرگوش کشی که در سوال قبل گفته شد، هر بار که مرد خرگوشی را بکشد که نه در سطر آن و نه در ستون ِ آن، خرگوش زنده ی دیگری نباشد، یک امتیاز می گیرد (دقت کنید که پس از کشتن خرگوش آخر نیز امتیاز می گیرد). او حداکثر چند امتیاز می تواند بگیرد؟

الف) 1 ب) 2 ج) 1384 د) 1385 ﻫ) 1386

جواب : گزینه ی د . می توان طوری این کار را کرد که در آخر مثلا همه ی خرگوش های روی قطر را کشت . (نامردا چقدر خشن ان)

m_honarmand_j
23-05-2007, 02:56
[ برای مشاهده لینک ، لطفا با نام کاربری خود وارد شوید یا ثبت نام کنید ]
20) یک رشته به طول 13 حرف،‌متشکل از حروف w و b را در نظر بگیرید. عمل به روز رسانی رشته بر حسب عدد i ( بین 1 و 13) را به این ترتیب تعریف می کنیم: از حرف i ام از سمت چپ شروع به کار می کنیم. آن حرف را تغییر می دهیم (یعنی w را به b و b را به w تغییر می دهیم). در صورتی که با این تغییر، حرف i ام از b به w تبدیل شود به سراغ حرف بعدی یعنی حرف سمت راستیش (در صورت وجود) رفته و همین کار را انجام می دهیم.


به عنوان مثال رشته ی bwbbbbwwbbwww بعد از انجام عمل به روز رسانی روی حرف چهارم، به رشته ی bwb[ برای مشاهده لینک ، لطفا با نام کاربری خود وارد شوید یا ثبت نام کنید ] تبدیل می شود. فرض کنید روی رشته ی [ برای مشاهده لینک ، لطفا با نام کاربری خود وارد شوید یا ثبت نام کنید ] عملیات زیر را به ترتیب انجام می دهیم:

1) یک بار عمل به روز رسانی روی حرف دوم.

2) یک بار عمل به روز رسانی روی حرف پنجم.

3) هشت بار عمل به روز رسانی روی حرف یکم.

4) بیست بار عمل به روز رسانی روی حرف ششم.

شما باید مشخص کنید که در نهایت تعداد b ها در رشته ی حاصل از مراحل بالا چند تاست.

الف) 9 ب) 10 ج) 4 د) 5 ﻫ) 3

جواب : گزینه ی د . اگر از چپ به راست هر مکان را به ترتیب 2 به توان 0 ، 1^2 ، 2^2 و ... 13^2 در نظر بگیریم و w را 0 و b را 1 در نظر بگیریم می توان با جمع در مبنای 2 به راحتی جواب را به دست آورد .

m_honarmand_j
23-05-2007, 02:59
دوستان نظری ، پیشنهادی ، انتقادی ، فهشی ، چیزی که حداقال امیدوار شم یکی هست که مطالب و می خونه .
(شاید اگه همین جوری پیش بره دیگه تو اینجا مطلبی نزارم . چون اصلا انگیزه ندارم . )

pp8khat
23-05-2007, 09:47
سلام.
آخه آقای هنرمند این مسئله هایی رو که تو میدی بیشتر به مسئله های المپیاد کامپوتر میخوره تا ریاضی
انقدر هو سخته که آدمو روانی میکنه.حداقل 2 تا سوال آسون توش میذاشتی ملت امیدوار بشن به خودشون.

pp8khat
23-05-2007, 09:52
مثل این مسئله:
معادله زیر را حل کنید.
x+y+z=0
x+y-z=2
x-y+z=2

m_honarmand_j
23-05-2007, 12:08
دوست عزیز pp8khat
اگر به عنوان تاپیک خوب نگاه کنید می بینید که عنوانش اتاق ترکیبیات ه و این سوالی که شما مطرح کردید هیچ ربطی به ترکیبیات نداره . همچنین سوالاتی که در این تاپیک مطرح می شود مربوط به ترکیبیات هستند . درضمن باید بگم که از شش سوال مرحله ی دوم المپیاد ریاضی دوتاش مربوط به ترکیبیات هست ، پس اینها فقط مربوط به کامپیوتر نمی شن . البته قبول دارم که سوالات ترکیبیات کامپیوتر با ریاضی تفاوت داره . (مثلا تو ریاضی بیشتر سوالات خفن مربوط به نظریه ی مجموعه هاست و کلا تیپ سوالاتش یک کم فرق داره . کلا ریاضی ها از ریاضیات کثیف استفاده می کنن) ولی هم اینکه من از سوالات ریاضی-اوی خوشم نمی یاد هم اینکه چون دارم واسه ی کامپیوتر می خونم بیشتر به سوالات کامپوتروی بر می خورم و به سوالات مربوط به ریاضی کمتر بر می خورم تا بتونم تو اینجا مطرح کنم . به همین دلیل ه که اصرار دارم دیگران هم سوال مطرح کنن .
در مورد سختی سوالات هم از این به بعد سعی می کنم در این بین سوالات آسون تر هم مطرح کنم (ولی خداییش کسی حتی اون آسوناش و هم حل نمی کنه دیگه)
بازم اگه نظری بود ، منتظرم .

hadi asiay
23-05-2007, 17:10
سلام
در هر یک از خانه های یک جدول n*n که در آن n فرد است یکی از اعداد 1 یا 1- را به دلخواه نوشته ایم . زیر هر ستون حاصلضرب همه ی عدد های این ستون و در سمت راست هر سطر حاصلضرب همه ی عدد های آن سطر را نوشته ایم . ثابت کنید مجموع همه ی این 2n حاصلضرب نمی تواند برابر صفر شود .
با سلام

با برهان خلف ثابت می کنیم
اگر مجموع همه ی 2nحاصلضرب صفر شود باید n تا از حاصل ضرب ها 1- وn تا از ان ها 1 شود پس حاصل ضرب همه ی این حاصلضرب ها برابر است با -1 به توان nضرب در 1 به توان 1 که چون nفرد است برابر با 1می شود
از طرفی این حاصل ضرب(حاصل ضرب همه ی سطر ها وستون ها) باید 1 شود چون که عدد مربوط به هر خانه دقیقا دوبار در حاصل ضرب ظاهر شده یک بار با ستون یک بار با سطر.
این تناقض است .

kaakaa
24-05-2007, 01:33
ممنون ولی من این سوالو گذاشته بودم واسه اونایی که ندیده بودن که یه ذره روش فکر کنن.

m_honarmand_j
24-05-2007, 14:48
با سلام

با برهان خلف ثابت می کنیم
اگر مجموع همه ی 2nحاصلضرب صفر شود باید n تا از حاصل ضرب ها 1- وn تا از ان ها 1 شود پس حاصل ضرب همه ی این حاصلضرب ها برابر است با -1 به توان nضرب در 1 به توان 1 که چون nفرد است برابر با 1می شود
از طرفی این حاصل ضرب(حاصل ضرب همه ی سطر ها وستون ها) باید 1 شود چون که عدد مربوط به هر خانه دقیقا دوبار در حاصل ضرب ظاهر شده یک بار با ستون یک بار با سطر.
این تناقض است .

اثبات درسته . ممنون آقای hadi asiay .

m_honarmand_j
24-05-2007, 14:51
ممنون ولی من این سوالو گذاشته بودم واسه اونایی که ندیده بودن که یه ذره روش فکر کنن.

ببخشید . من یه جورایی احساس کردم سوال و گذاشتی من حل کنم . از دفعه ی بعد اول چند روز سب می کنم ، اگه کسی جواب نداد ، اونوقی جواب می دم . اگر هم دیدید که بعد از چند روز من هم جواب ندادم لطفا خودتون جوابشو بگید .

m_honarmand_j
24-05-2007, 14:55
[ برای مشاهده لینک ، لطفا با نام کاربری خود وارد شوید یا ثبت نام کنید ]

21) عباس می خواهد شکل 1 را با 6 قطعه از قطعاتی که در شکل 2 نشان داده شده بپوشاند،‌به طوری که درست یک شش ضلعی پوشیده نشده در آن باقی بماند. او به چند طریق می تواند این کار راانجام دهد؟



الف) 1 ب) 2 ج) 4 د) 3 ﻫ) 8

جواب : گزینه ب جواب ضحیح ه . اگر شکل و با سه رنگ بپوشونید به طوری که هیچ دو رنگی کنارهم نباشند اونوقت می بینید که یک رنگ یک بار بیشتر از بقیه ظاهر شده . پس خونه ی خالی باید اونرنگی باشه . با یک کم چک کردن می بینید که فقط خونه ی وسط می تونه خالی بمونه . برای پر کردن بقیه ی جا ها هم به دو طریق می تونیم این کار رو بکنیم . (یکی از گوشه ها رو در نظر بگیرید ، جوابش در می یاد)

hadi asiay
26-05-2007, 14:18
سلام
6 نفر در یک مهمانی حضور دارند احتمال اینکه حداقل دو نفر در یک روز هفته متولد شده با شند چیست؟
اول تعداد حالت هایی که همه در روز های متفائتی متئلد سوند راحساب میمنیم بهد این مقدار رااز یک کم میکنیم
کلن 7^6 حات یبرای تولد افراد وجود داررد که در7*6*5*...2حالت روز های تولد متفاوتی دارند
پس احتمال اینکه در روز های متفاوتی متولد شوند براراست با!7تقسیم بر7^6
حالا احتمال تولد در روزهای متفائت برابر است با 1 منهای عدد بالا

hadi asiay
26-05-2007, 14:22
سلام
6 نفر در یک مهمانی حضور دارند احتمال اینکه حداقل دو نفر در یک روز هفته متولد شده با شند چیست؟
اول تعداد حالت هایی که همه در روز های متفاوتی متولد شوند راحساب میکنیم بعد این مقداررا از 1کم میکنیم.
کلن 7^6 حالت برای تولد افراد وجود داررد که در7*6*5*...2حالت روز های تولد متفاوتی دارند
پس احتمال اینکه در روز های متفاوتی متولد شوند براراست با!7تقسیم بر7^6
حالا احتمال تولدحداقل دونفردریک روز برابر است با 1 منهای عدد بالا

hadi asiay
26-05-2007, 14:25
حل اولی ویرایش نشده بود ببخشین!

m_honarmand_j
27-05-2007, 16:02
سلام
یک جدول m*n شطرنجی داریم . در هر حرکت می تونیم یک مستطیل(و نه یک مربع) را انتخاب کرده و رنگ همه ی خانه های اونو عوض کنیم . به ازای کدام n ها و m ها این کار امکان پذیر است؟

m_honarmand_j
27-05-2007, 16:04
22) آیدین یک ماشین حساب یک رقمی دارد که با 7 لامپ باریک و دراز،‌هر یک از ارقام 0 تا 9 و علامت «-» (به معنای خطای محاسبه) را مطابق شکل زیر نشان می دهد.



متأسفانه به دلیل فرسودگی ماشین حساب، همه ی 7 لامپ آن هم زمان سوخته اند. اکنون آیدین می خواهد k تا از این لامپ ها را طوری با لامپ های سالم عوض کند که بتواند با دیدن روشن یا خاموش بودن لامپ های سالم، پی به مقدار دقیق ماشین حساب ببرد. دقّت کنید که اگر یک لامپ سالم در ماشین حساب قرار داده شود، دیگر نمی توان جای آن را تغییر داد. حداقل مقدار k برای این منظور چند است؟

الف) 3 ب) 4 ج) 5 د) 6 ﻫ) 7

جواب : در پاسخنامه ای که سایت باشگاه داده گزینه ی ج جواب ه . ولی من با کمتر از 6 تا نتونستم این کار و بکنم . اگر کسی تونست به من هم بگه.

m_honarmand_j
27-05-2007, 16:19
23) در ابتدا یک مثلث متساوی الاضلاع داریم که طول اضلاعش برابر 4 است (مانند شکل سمت چپ).

در هر مرحله می توانیم یک تکه از شکل باقی مانده که شروط زیر را دارد بِکنیم و دور بریزیم.

* تکه باید یک مثلث متساوی الاضلاع باشد.

* اضلاع تکه باید روی خطوط کشیده شده در شکل باشد.

* با حذف کردن این تکه از شکل نباید هیچ سوراخی در آن ایجاد شود. (یعنی باید حداقل یکی از اضلاع تکه ای که می خواهیم حذف کنیم، کاملا مجاور ناحیه ی بیرونی باشد.)

بدیهی است در صورتی که شکل باقی مانده یک مثلث متساوی الاضلاع باشد می توان در یک مرحله همه ی آن را دور ریخت. در شکل زیر یک مثال تا دو مرحله نشان داده شده است.



می خواهیم در k مرحله کل شکل اولیه را دور بریزیم. این کار به ازای چند تا از مقادیر زیر به عنوان ِ‌مقدار ِ k، امکان پذیر است؟

4، 5، 7، 10، 14

الف) 1 ب) 2 ج) 3 د) 4 ﻫ) 5

جواب : گزینه ی ج . اگر دقت کنید اعدادی که می تونن جواب باشن به صورت 3k+1 هستند یعنی 4 ، 7 و 10 .

m_honarmand_j
27-05-2007, 16:20
از آقای hadi asiay هم به خاطر شرکتشون در بحث ممنون ام .
(می خواستم بگم دفعه ی بعد اگر دیدید متنتون ویرایش نشده می تونید همون متن و ویرایش کنید . )

hadi asiay
29-05-2007, 12:56
سلام
یک جدول m*n شطرنجی داریم . در هر حرکت می تونیم یک مستطیل(و نه یک مربع) را انتخاب کرده و رنگ همه ی خانه های اونو عوض کنیم . به ازای کدام n ها و m ها این کار امکان پذیر است؟
اگر منظورتون اینه که همه ی خونه ها همرنگ بشن برای همه ی جدول ها غیر از 1 در 1و 2 در1 این کار امکان پذیره زیرا که جدول 1 در 3 و 1در4 و1در 5رو میشه همرنگ کرد (خودتون ببینین!) وبعدشم هر جدول mدرnرو میتونین به ردیف ها ی1درn تبدیل کر د جدول nدر1 رو هم میشه به تکه های 1در 3 ی یا 1در 4 یا 1در5 ی تقسیم کرد بعد هم کل ردیفو هم رنگ کرد بعدم همه ی ردیفارو همرنگ میکنیم

hadi asiay
29-05-2007, 12:58
سلام
یک جدول m*n شطرنجی داریم . در هر حرکت می تونیم یک مستطیل(و نه یک مربع) را انتخاب کرده و رنگ همه ی خانه های اونو عوض کنیم . به ازای کدام n ها و m ها این کار امکان پذیر است؟
اگر منظورتون اینه که همه ی خونه ها همرنگ بشن برای همه ی جدول ها غیر از 1 در 1و 2 در1 این کار امکان پذیره زیرا که جدول 1 در 3 و 1در4 و1در 5رو میشه همرنگ کرد (خودتون ببینین!) وبعدشم هر جدول mدرnرو میتونین به ردیف ها ی1درn تبدیل کر د جدول nدر1 رو هم میشه به تکه های 1در 3 ی یا 1در 4 یا 1در5 ی تقسیم کرد بعد هم کل ردیفو هم رنگ کرد در نهایت همه ی ردیفارو همرنگ میکنیم

hadi asiay
29-05-2007, 13:03
سلام
یک جدول m*n شطرنجی داریم . در هر حرکت می تونیم یک مستطیل(و نه یک مربع) را انتخاب کرده و رنگ همه ی خانه های اونو عوض کنیم . به ازای کدام n ها و m ها این کار امکان پذیر است؟
اگر منظورتون اینه که همه ی خونه ها همرنگ بشن برای همه ی جدول ها غیر از 1 در 1و 2 در1 این کار امکان پذیره زیرا که جدول 1 در 3 و 1در4 و1در 5رو میشه همرنگ کرد (خودتون ببینین!) وبعدشم هر جدول mدرnرو میتونین به ردیف ها ی1درn تبدیل کر د جدول nدر1 رو هم میشه به تکه های 1در 3 ی یا 1در 4 یا 1در5 ی تقسیم کرد بعد هم کل ردیفو هم رنگ کرد بعدم همه ی ردیفارو همرنگ میکنیم

m_honarmand_j
10-06-2007, 12:05
سلام آقای hadi asiay
بله منظورم همرنگ کردن همه ی خونه ها بود .
(لطفا در فرستادن جواب بیشتر دقت کنید و یک مطلب رو چند بار نفرستید)
این سوال و گزاشتم چون بعضی ها گفتن سوالا سخت هستند . خب این آسون بود ولی جز یک نفر کسه دیگه ای جواب نداد.!!

m_honarmand_j
10-06-2007, 12:23
دوستان چند وقته که مطلب جدید نذاشتم . چون فعلا زمان امتحانات ه . انشااللاه بعد از امتحانات جبران می کنم .

m_honarmand_j
14-06-2007, 19:51
سلام
25) در شکل زیر 27 نخود از بالا به سمت پایین انداخته می شود. نخود ها به سمت پایین حرکت می کنند تا درون یکی از مربع ها قرار بگیرند. درون هر دایره یک علامت یا قرار دارد که در حالت عادی دیده نمی شود. با توجه به جهت علامت یک دایره، نخود پس از ورود به آن دایره به سمت «پایین سمت راست» یا «پایین سمت چپ» حرکت می کند. ما نمی توانیم در حالت عادی جهت علامت های دایره ها یا تعداد نخود های موجود در مربع ها و دایره ها را ببینیم.


عمل «تغییر جهت» به این صورت تعریف می شود: به یکی از دایره ها از نزدیک نگاه می کنیم و علامت قرار داده شده در آن را می بینیم و اگر خواستیم آن را تغییر می دهیم.

ما می توانیم هر موقع که خواستیم انداختن نخود ها را متوقف کنیم و عمل «تغییر ِ جهت» را به تعداد دلخواه انجام دهیم و دوباره انداختن نخود ها را ادامه دهیم.



می خواهیم تعدادی عمل «تغییر جهت» انجام دهیم، به طوری که وقتی همه ی 27 نخود افتادند، در هر مربع دقیقا به تعداد عددی که روی آن نوشته دشه نخود قرار بگیرد. حداقل چند عمل «تغییر جهت» نیاز داریم به طوری که به هر نحوی که علامت ها در ابتدا جهت دهی شده باشند، بتوانیم این کار را انجام دهیم؟

الف) 6 ب) 7 ج) 11 د) 12 ﻫ) 14

جواب : به سادگی می توان دید که هر دایره را باید دوبار دید . پس 12 بار باید این عمل را انجام دهیم . پس جواب گزینه ی د است .

m_honarmand_j
14-06-2007, 19:52
سلام
26) در مسئله ی قبل، فرض کنید وقتی در یک مربّع به تعداد ِ عددی که روی آن نوشته شده نخود قرار بگیرد،‌از پُر شدن آن جعبه مطّلع می شویم.



حال با توجه به داشتن این امکان اضافی بگویید، در شکل روبرو، حداقل چند عمل «تغییر جهت» لازم است، تا جهت اوّلیه ی علامت ها هر چه که باشد،‌بتوانیم به هر مربّع دقیقا به تعداد عددی که رویش نوشته شده نخود بفرستیم؟

الف) 7 ب) 8 ج) 12 د) 13 ﻫ) 14

جواب : این بار هر دایره را باید یک بار ببینیم پس به 7 بار انجام عمل نیاز داریم . پی جواب گزینه ی الف است .

m_honarmand_j
14-06-2007, 19:56
سلام
27) ده شرکت تولید کننده ی کنسرو مورد آزمایش کیفیت قرار گرفتند. می دانیم که دقیقا یکی از شرکت ها از گوشت فاسد استفاده می کند. آزمایش به این صورت بود که به هر داوطلب، دو کنسرو از دو شرکت مختلف داده شد و به هیچ دو نفری جفت کنسرو های یکسانی داده نشد. می دانیم اگر کسی کنسرو فاسد بخورد، می میرد. پس از این که داوطلب ها کنسرو هایشان را خوردند، اطلاعات بدست آمده از مرگ داوطلبان برای یافتن شرکت متخلف کافی نبود. تعداد داوطلب ها حداکثر چند نفر بوده است؟

الف) 20 ب) 28 ج) 29 د) 36 ﻫ) 45

جواب : گزینه ی د است . انتخاب 2 از 9 تا جفت کنسرو داریم که از کارخانه ای که گوشت فاسد استفاده می کند هیچ کنسروی انتخاب نکرده باشیم . یعنی 36 تا . پس تعداد داوطلب ها حداکثر 36 تا است و اگر بیشتر از این باشد حداقل یک کنسرو از شرکت مورد نظر انتخاب می شود و ما می توانیم شرکت را پیدا کنیم .

m_honarmand_j
14-06-2007, 19:59
سلام
28) پینوکیو برای یافتن پدر ژِپِتو وارد شکم نهنگ شد. شکم نهنگ به شکل مجموعه ای از سه راهی ها است که هر کدام به سه سه راهی ِ دیگر متصل است. فاصله ی دو سه راهی متصل 1 متر است. پدر ژپتو نوشته شده است. پینوکیو در یکی از این سه راهی هاست که تابلوی آن 1385 را نشان می دهد. با این فرض که پینوکیو به اندازه ی کافی باهوش است، در بدترین حالت، چند متر باید راه برود تا به پدر ژپتو برسد؟

الف) 1385 ب) 2771 ج) 4155 د) 4157 ﻫ) 6925

جواب: مشاهده می شود که بجز اولین حرکت به ازای حداکثر هر سه متر می تواند یک متر خود را نزدیکتر کند و در حرکت اول به ازای حداکثر 5 متر می تواند یک متر خود را نزدیکتر کند . پس جواب برابر ایت با :
5+ 3*1384 = 4157
پس گزینه ی د صحیح است .

m_honarmand_j
14-06-2007, 20:02
سلام
29) در شکل روبرو 8 شهر و راه های ارتباطی آن ها نمایش داده شده است. نیلوفر و لی لی می خواهند این شهر ها را ببینند و با هم قرار گذاشته اند که عوارض ورود به شهر ها را یکی در میان بپردازند.



نیلوفر یکی از شهر ها را برای شروع مسافرت انتخاب می کند و عوارض ورود به آن را می پردازد. از این به بعد، ابتدا لی لی و سپس نیلوفر، هر کدام در نوبت خود شهر مجاوری (شهری که به شهر فعلی راه مستقیم دارد) که هنوز ندیده اند را انتخاب می کند. سپس آن ها به شهر مسافرت می کنند و کسی که شهر جدید را انتخاب کرده، عوارض آن را پرداخت می کند. آن ها این کار را تا دیده شدن همه ی شهر ها ادامه می دهند. هر کدام از این دو نفر می خواهد هزینه ی کمتری از دیگری بپردازد.

نیلوفر و لی لی قبل از آن که مقادیر عوارض شهر ها را در کتابچه ی راهنمای سفر خود ببینند، حرف های زیر را زده اند. کدام یک از این گفته ها به ازای مقادیر مختلفی که ممکن است در دفترچه ی راهنما نوشته شده باشد همواره درست است؟

الف) نیلوفر: من کمتر خرج خواهم کرد.

ب) لی لی: من کمتر خرج خواهم کرد.

ج) نیلوفر: ممکن است مقادیر دفترچه ی راهنما طوری باشد که من مجبور شوم بیشتر خرج کنم.

د) لی لی: مجبور نیستم بیشتر خرج کنم.

ﻫ) نیلوفر: مجبور نیستم بیشتر خرج کنم.

جواب : مشاهده می شود که شک یک گراف دوبخشی است (از هر بخش مجبوریم به بخش دیگر برویم) . در نتیجه نفر اول می تواند مجموع عوارض هر دسته را حساب کند و آن دسته که بصرفه تر است را انتخاب کند . در نتیجه گزینه ی ه صحیح است .

FerineFast
01-07-2007, 21:07
آقای هنرمند عزیز
بهتر نسیت که کل جواب ها را در یک فایل pdf ارائه دهید ؟!

m_honarmand_j
21-07-2007, 02:39
سلام بر همه ی دوستان
چند مدت نبودم ولی الان هستم .
می خواستم به اطلاع برسونم که دیگه من پستی نخواهم فرستاد .
بعد از اینکه در المپیاد کامپیوتر قبول نشدم(حالا بماند که چرا قبول نشدم ، چون واقعا برای خودم هم عجیب بود) در مدرسه هم نمره ی 5/14 رو در درس ریاضی گرفتم تا نتونم سال دیگه تو المپیاد ریاضی شرکت کنم(باید نمره ی حداقل 15 و می گرفتم) . خلاصه همه ی عوامل دست به دست هم دادند تا من دیگه از صحنه برم بیرون .
به همین جهت من هم رفتم سراغ کار خودم و به برنامه نویسی مشغول شدم . (البته من خودم به علت علاقه ای که به گراف دارم روی اون کار می کنم)
چون اگه بخوام باز هم در این سایت مشغول باشم واقعا وقتم تلف می شه . چون نه کسی از این تاپیک بازدید می کنه و نه کسی نظر می ده و نه برای من فایده ای داره . پس چرا وقتم و بیهوده تلف کنم؟
ازتون ممنونم که در این مدت منو تحمل کردید و از آقای مفیدی (اگر این متن و یه زمانی خوندند) تشکر می اکن که منو با این سایت آشنا کرد (حالا بماند که این آشنایی برام خوب بوده یا نه).
اگر با من کاری داشتید و فکر می کردید که کاری ازم بر می آد می تونید بهم میل بزنید و یا برام off بزارید . (mostafa_o2@yahoo.com)
خداحافظ همگی .

ali_hp
23-07-2007, 02:34
سلام اقا مصطفی
راستش منم فکر می کردم قبول میشید!!!
توی این تاپیک کلی مساله خوب ترکیبیات هست+راه حل مسایل مرحاه اول المپیاد کامپیوتر امسال که میتونه مورد استفاده خیلی ها قرار بگیره.و با کمی سر و سامون دادن خیلی بهتر هم میشه.
به هر حال فعالیت شما تو این مدت اصلان وقت تلف کردن نبوده.وخیلی هم مفید بوده.
اون نیم نمره رو اگه با مدرستون هماهنگ کنید احتمال زیاد حل میشه!البته نمیدونم واقعا حل شدنش خوبه یا بد!

eh_mn
23-07-2007, 10:53
سلام بر همه ی دوستان
چند مدت نبودم ولی الان هستم .
می خواستم به اطلاع برسونم که دیگه من پستی نخواهم فرستاد .
بعد از اینکه در المپیاد کامپیوتر قبول نشدم(حالا بماند که چرا قبول نشدم ، چون واقعا برای خودم هم عجیب بود) در مدرسه هم نمره ی 5/14 رو در درس ریاضی گرفتم تا نتونم سال دیگه تو المپیاد ریاضی شرکت کنم(باید نمره ی حداقل 15 و می گرفتم) . خلاصه همه ی عوامل دست به دست هم دادند تا من دیگه از صحنه برم بیرون .
به همین جهت من هم رفتم سراغ کار خودم و به برنامه نویسی مشغول شدم . (البته من خودم به علت علاقه ای که به گراف دارم روی اون کار می کنم)
چون اگه بخوام باز هم در این سایت مشغول باشم واقعا وقتم تلف می شه . چون نه کسی از این تاپیک بازدید می کنه و نه کسی نظر می ده و نه برای من فایده ای داره . پس چرا وقتم و بیهوده تلف کنم؟
ازتون ممنونم که در این مدت منو تحمل کردید و از آقای مفیدی (اگر این متن و یه زمانی خوندند) تشکر می اکن که منو با این سایت آشنا کرد (حالا بماند که این آشنایی برام خوب بوده یا نه).
اگر با من کاری داشتید و فکر می کردید که کاری ازم بر می آد می تونید بهم میل بزنید و یا برام off بزارید . (mostafa_o2@yahoo.com)
خداحافظ همگی .

سلام دوست عزیز

نمیدونم می دونی یا نه. من دانشجوی سال چهارم رشته ریاضی هستم و تا چند روز دیگه فارغ التحصیل میشم و اگه راستشو بخوای علت اینکه تو این تاپیک پست نمی دادم این بود که بلد نبودم سوالا رو حل کنم و همیشه شما رو بخاطر اینکه اینقدر راحت مسئله حل می کنید تحسین می کردم. به هر حال شما هر تصمیمی بگیری به خودت مربوطه ولی به نظر من اگر قرار باشه دیگه اینجا پست ندی خیلی حیف میشه چون این تاپیک رو تا اینجا رسوندی و فقط خودت می تونی کاملترش کنی.
برای بهتر شدن این تاپیک من فکر می کنم بهتره از روش های حل مسئله بیشتر صحبت کنی.

امیدوارم هر کجا که هستی موفق باشی.

pp8khat
18-08-2007, 20:15
سلام.
لطفاً از این به بعد عبارت هایی رو که تجزیشون جالب و سخت هستش(مخصوصاً افزودن و کاستن ها) رو بذارید تا همه استفاده کنن(در سطح دانشگاه هم مهم نیست می تونین بذارین)....
من خودم 10 تا درست کردم(البته یکی مونده به آخری رو از مبتکران زدم!)

جواب های هر کدوم رو که بلد نبودید من در خدمتم(به قول امیر جان اظهار فضل!!!)
این مثال های من هم از بالا به پایین سطحشون بالا می ره و سخت تر می شن...
لطفاً تا آخرین حد ممکن عبارات رو تجزیه کنید!مثلاً تو تجزیه ی x^4-16 تنها به (x^2-4)(x^2+4)بسنده نکنید و x^2-4 رو هم به (x-2)(x+2) تجزیه کنید...
(البته دیگه نمی خواد" n بازی"(!)در بیارید یعنی دیگه x^2-2 رو ننویسید (x-2^1/2)(x+2^1/2) )و....
من هم هر 1 یا 2 هفته یه بار یه 10 تای دیگه هم میذارم...
PostScript(!):
(برای راهنمایی ها)
منظور از تجزیه یک عبارت جبری یعنی آن را به صورت ضرب یا تقسیم چند عامل تغییر ظاهر دادن.
مثال)
(x-1).(x-1)=x^2-2x+1
*علامت ^ برای نشان دادن بالا نوشت (محل نوشتن توان یا بار الکتریکی یون ها یا ظرفیت لایه ها)به کار می رود.یعنی5^2=32
پیشاپیش از همکاری شما کمال تشکر را دارم(!)
(زبان فارسی 20 شدما!!!)
[ برای مشاهده لینک ، لطفا با نام کاربری خود وارد شوید یا ثبت نام کنید ]

pp8khat
21-08-2007, 22:24
باشه دیگه...
همکاری نمی کنیدووو...
دیگه به من محل نمی ذارینووو...
خیالی نیست...
تا وقتی که این خرزو خان هست من نیازی به همکاری شماها ندارم(!)...
تازه خرزو خان عزیز وقتی که عبارات رو تجزیه می کنه،جوابشو بهم می فرسته تا مطمئن بشه درست حلش کرده...(دانش آموز پر تلاشیه...مرحبا(!)) 5 6 تا تجزیه هم بهم داده...
اینم 10 تا تجزیه دیگه برای دوستان همکار و محل گذارم(!!)...
(بعضی ها که جلوشون علامت سواله باید حاصل عبارت رو به دست بیارید...)
(سوال هفتم رو با کمی تغییرات از مبتکران برداشتم(سوال تربیت معلم) و سوال نهم هم با اندکی تغییرات از آزمون مرآت...بقیه تالیف خودم هستش...چقدر خودمو تحویل می گیرماااا!!!)
[ برای مشاهده لینک ، لطفا با نام کاربری خود وارد شوید یا ثبت نام کنید ]

eh_mn
21-08-2007, 23:58
سلام.
اگه ممكنه براي سوال‌ها شماره بذار كه راحتتر بشه در موردشون صحبت كرد.
اينم يك معادله كه با استفاده از تجزيه ميشه حلش كرد:

[ برای مشاهده لینک ، لطفا با نام کاربری خود وارد شوید یا ثبت نام کنید ]

موفق باشي.

pp8khat
25-08-2007, 20:42
سلام.
اگه ممكنه براي سوال‌ها شماره بذار كه راحتتر بشه در موردشون صحبت كرد.
اينم يك معادله كه با استفاده از تجزيه ميشه حلش كرد:

[ برای مشاهده لینک ، لطفا با نام کاربری خود وارد شوید یا ثبت نام کنید ]

موفق باشي.

سلام آقا احسان!
نمی دونی باچه بدبختی معادلتو حل کردم...
از عدد گذاشتن بگیر تا نمودار درجه 4 کشیدن!!
لطفاً راه تجزیه(اورژینالشون!)بذارید...
ممنون
حل:
[ برای مشاهده لینک ، لطفا با نام کاربری خود وارد شوید یا ثبت نام کنید ]

eh_mn
25-08-2007, 22:24
سلام آقا احسان!
نمی دونی باچه بدبختی معادلتو حل کردم...
از عدد گذاشتن بگیر تا نمودار درجه 4 کشیدن!!
لطفاً راه تجزیه(اورژینالشون!)بذارید...
ممنون

با سلام.
خيلي قشنگ حل كردين.
اينم راه حل (بقول خودتون اورجينالش!!)


[ برای مشاهده لینک ، لطفا با نام کاربری خود وارد شوید یا ثبت نام کنید ]

با فرض [ برای مشاهده لینک ، لطفا با نام کاربری خود وارد شوید یا ثبت نام کنید ] و حل معادله با اين متغير جديد داريم


[ برای مشاهده لینک ، لطفا با نام کاربری خود وارد شوید یا ثبت نام کنید ]

m_honarmand_j
06-09-2007, 01:10
دوباره سلام

من به زودی بر خواهم گشت و فعالیتم رو در این تاپیک از سر می گیرم .

این بار با کلی تغییرات و نحوه ای جدید .

پس منتظر باشید .

همچنین از تمام کسانی که مایل به همکاری هستند می خوام که آمادگی خودشون و اعلام کنن .

(می دونم الان بعضی ها با خودشون می گن این یارو رو نیگا . فقط می خواست کلاس بزاره و یه مدت مارو بپیچونه . ولی این طور نیست . در اصل با صحبت هایی که آقای مفیدی باهام داشتن تصمیم گرفتم که دوباره مشغول به کار بشم ولی نه مثل سابق . پس از دیگران هم می خواهم که به من کمک کنن تا بتونیم با هم یه کار مفید انجام بدیم . )


فعلان تا وقتی که دوباره برگردم .
(به زودی بر می گردم)

pp8khat
06-09-2007, 11:46
دوباره سلام

من به زودی بر خواهم گشت و فعالیتم رو در این تاپیک از سر می گیرم .

این بار با کلی تغییرات و نحوه ای جدید .

پس منتظر باشید .

همچنین از تمام کسانی که مایل به همکاری هستند می خوام که آمادگی خودشون و اعلام کنن .

(می دونم الان بعضی ها با خودشون می گن این یارو رو نیگا . فقط می خواست کلاس بزاره و یه مدت مارو بپیچونه . ولی این طور نیست . در اصل با صحبت هایی که آقای مفیدی باهام داشتن تصمیم گرفتم که دوباره مشغول به کار بشم ولی نه مثل سابق . پس از دیگران هم می خواهم که به من کمک کنن تا بتونیم با هم یه کار مفید انجام بدیم . )


فعلان تا وقتی که دوباره برگردم .
(به زودی بر می گردم)
فقط سوالای سخت نذار که دوباره روانی نشم:2::2:
چند تا سوال هم از المپیاد ریاضی بذاری بد نیست

m_honarmand_j
09-09-2007, 16:11
سلام به همه ی دوستان
من هنوز منتظر هستم که برای همکاری اعلام آمادگی کنید .
من تصمیم گرفتم که آموزش تر کیبیات و از اولش شروع کنم و تا اونجا که سوادم بکشه پیش ببرم .
به همین دلیل ممکنه یک مدت برای اونایی که قبلا این مطالب و خوندن و می دونن کسالت آور باشه . ولی برای کسانی که قبلا مطالعه ای در باره ی ترکیبیات ندارند می تونه جالب باشه . همچنین تمام کسانی که می خواهند خود را برای المپیاد ریاضی و کامپیوتر آماده کنند نیز مفید می باشد ، چون اگر به هر کلاسی برای این منظور بروند اول از همه مجبورند که این مطالب رو یاد بگیرند . همچنین سعی می کنم که مطالب پایه رو سریع بگم و به جاهای جالب برسم .
در بین آموزش هم از مثال استفاده می کنم و برای هر بخش نیز تمریناتی را نیز می گذارم که اگر کسی از اول آموزش من را همراهی کند می تواند اکثر سوالات را حل کند . همچنین ممکن است جواب تمام سوالات را خودم نگذارم و تنها جواب تمریناتی را که فکر کنم مفیدتر هستند و همچنین تمریناتی که دیگران درخواست کنند را بدهم .
مشکل اصلی که الان با آن روبه رو هستم این است که باید برای این تاپیک یک فهرست تهیه کنم که خودم وقت نمی کنم این کار رو انجام بدهم و نمی دونم باید چیکار کنم .
فعلا نکته ی دیگری به ذهنم نمی رسه . اگه بعدا نکته ای به خاطرم رسید می گم .
پس فعلا .

m_honarmand_j
09-09-2007, 16:17
دوست عزیز pp8khat
من زیاد سوالات المپیاد ریاضی به پستم نمی خوره (البته فعلا) ولی اگر به پستم خورد حتما این کار و می کنم .
همچنین از این به بعد چون از پایه شروع به گفتن مطالب می کنم سعی می کنم که در هر بخش آموزش تمرینات در سطوح مختلف بزارم و البته احتمالا تمرینات مربوط به بخش های ابتدایی خیلی ساده باشند ولی کم کم که پیش بریم هم مطالب و سخت تر می کنم و هم تمرینات و هم مساثل و .

m_honarmand_j
09-09-2007, 16:21
j آآموزش ترکیبیات 1-1
اصل جمع : فرض کنید k یک عدد طبیعی و A1 و A2 و ... و Ak ، k مجموعه ی متناهی و دو به دو مجزا باشند ؛ یعنی به ازای هر i , j = 1,2,3,…,k ، i≠j ، داشته باشیم Ai∩Aj=Ǿ . آنگاه داریم :
|Úi=1,..k Ai| =|A1ÚA2Ú…ÚAk| = ∑ki=1|Ai|
این اصل جمع به صورت مجموعه ای بود . به طور ساده تر می توان گفت که اگر برای انجام کاری k انتخب مختلف داشته باشیم A1) و A2 و ... و (Ak به طوری که هیچ کدام از آن ها با دیگری اشتراک نداشته باشند (Ai∩Aj=Ǿ) آنگاه برای آنکه بدانیم در مجموع برای انجام آن کار چند حالت مختلف داریم باید تعداد حالت های موجود برای هر انتخاب را با یکدیگر جمع بکنیم .
مثال 1-1-1: فرض کنید می خواهیم از شهر A به شهر B برویم . برای این کار می توانید از راه هوایی ، دریایی ، زمینی و ریلی استفاده کنیم . پس A1 راه هوایی ، A2 راه دریایی ، A3 راه زمینی و A4 راه ریلی می باشد . فرض کنید برای راه هوایی 2 مسیر مختلف داشته باشیم . برای راه دریایی 1 مسیر . برای راه زمینی 4 مسیر و برای راه ریلی 2 مسیر مختلف وجود داشته باشد .
یعنی : |A1| = 2 , |A2| = 1 , |A3| = 4 , |A4| = 2
می خواهیم بدانیم که به چند طریق مختلف می توانیم خود را از شهر A به شهر B برسانیم ؟
به دلیل آنکه حالت های ما (هوایی ، دریایی ، زمینی و ریلی) از هم مجزا هستند و با هم اشتراکی ندارند پس باید از اصل جمع استفاده کنیم . در نتیجه ما می توانیم به 2+1+4+2 = 9 طریق کار مورد نظر را انجام دهیم .

تمرین 1-1-1: علی 2 نوع پیراهن آبی ، 3 نوع پیراهن سبز ، 1 نوع پیراهن سفید و 3 نوع پیراهن سیاه دارد . او به چند طریق می تواند یک پیراهن برای پوشیدن انتخاب کند ؟

m_honarmand_j
09-09-2007, 16:26
ترکیبیات 1-2
اصل ضرب : اگر k یک عدد طبیعی و A1 ،A2 ،... وAk ، k مجموعه ی متناهی باشند ؛ آنگاه داریم :
|∏ki=1 Ai| = ∏ki=1|Ai| = |A1| * |A2| * … * |Ak|
مثال1-2-1: ما می خواهیم از شهر A به شهر D برویم و برای این کار ابتدا باید از شهر A به شهر B ، سپس از شهر B به شهر C و در آخر از شهر C به شهر D برویم . اگر از شهر A به 2 طریق بتوان به شهر B رفت ( |A1| =2 ) ، از شهر B به 3 طریق بتوان به شهر C رفت (|A2| = 3 ) و از شهر C بتوان به 5 طریق به شهر D رفت (|A3| = 5 ) آنگاه ما می خواهیم بدانیم که در کل به چند طریق می توانیم خود را از شهر A به شهر D برسانیم ؟
طبق اصل ضرب جواب برابر خواهر بود با 30 = 5*3*2 .

تمرین 1-2-1: علی 5 نوع شلوار ، 10 نوع پیراهن ، 8 نوع کراوات و 2 نوع ساعت دارد . او به چند طریق می تواند لباس بپوشد ؟

m_honarmand_j
09-09-2007, 16:33
ترکیبیات 1-3

اصل متمم : اگر A یک زیرمجموعه ی ، مجموعه ی متناهی مرجع U باشد ، آنگاه داریم :
|A'| = |U| - |A|

m_honarmand_j
09-09-2007, 16:35
ترکیبیات 2-1

اصل شمارش : فرض کنید قرار است دو آزمایش انجام شود . در این صورت اگر آزمایش (1) به هر یک از m برآمد ممکن منتهی شود و اگر برای هر برآمد آزمایش (1) ، n برآمد ممکن از آزمایش (2) وجود داشته باشد ، آنگاه برای دو آزمایش جمعا mn برآمد وجود دارد .
اثبات اصل اساسی : این اصل اساسی را می توان با شمارش کلیه ی بر آمد های ممکن دو آزمایش به صورت زیر ثابت کرد :
(1,1),(1,2),…,(1,n)
(2,1),(2,2),…,(2,n)
.
.
.
(m,1),(m,2),…(m,n)
که در آن می گوییم برآمد (i,j) است ، اگر آزمایش (1) به برآمد ممکن i و سپس آزمایش (2) به برآمد ممکن j منجر شود ، پس مجموعه ی برآیند ممکن عبارت است از m سطر که هر سطر شامل n عنصر است که در نتیجه حکم ثابت می شود .


مثال 2-1-1: گروه کوچکی متشکل از ده مرد است که هر یک از آنها سه فرزند پسر دارد . اگر قرار باشد یک مرد و یکی از پسرانش را به عنوان پدر و پسر سال انتخاب کنند ، چند انتخاب مختلف وجود دارد ؟
جواب : اگر انتخاب این مرد را به عنوان برآمد آزمایش نخست و انتخاب بعدی یکی از پسرانش را به عنوان برآمد آزمایش دوم در نظر بگیریم ، از اصل اساسی نتیجه می شود که 30 = 10 * 3 انتخاب ممکن وجود دارد .

m_honarmand_j
09-09-2007, 16:39
تمرین 2-1-1: چند عدد یک رقمی و چند عدد دو رقمی وجود دارد ؟

تمرین 2-1-2: چند عدد دو رقمی بدون تکرار ارقام وجود دارد ؟

تمرین 2-1-3: چند عدد دو رقمی فرد وجود دارد ؟

تمرین 2-1-4: چند عدد دو رقمی زوج وجود دارد ؟

تمرین 2-1-5: چند عدد دو رقمی فرد بدون تکرار ارقام وجود دارد ؟

تمرین 2-1-6: چند عدد دورقمی زوج بدون تکرار ارقام وجود دارد ؟

تمرین 2-1-7: دو آزمایش قرار است انجام شود . اولی می تواند به هر یک از n برآمد ممکن منجر شود . اگر آزمایش اول به نتیجه ی شماره ی i منتهی شود ، آن گاه آزمایش دوم می تواند به هر یک از ni برآمد ممکن منتهی شود i=1,2,..,m . تعداد برآمد های ممکن دو آزمایش چند تا است ؟

pp8khat
09-09-2007, 16:45
سلام به همه ی دوستان
من هنوز منتظر هستم که برای همکاری اعلام آمادگی کنید .
من تصمیم گرفتم که آموزش تر کیبیات و از اولش شروع کنم و تا اونجا که سوادم بکشه پیش ببرم .
به همین دلیل ممکنه یک مدت برای اونایی که قبلا این مطالب و خوندن و می دونن کسالت آور باشه . ولی برای کسانی که قبلا مطالعه ای در باره ی ترکیبیات ندارند می تونه جالب باشه . همچنین تمام کسانی که می خواهند خود را برای المپیاد ریاضی و کامپیوتر آماده کنند نیز مفید می باشد ، چون اگر به هر کلاسی برای این منظور بروند اول از همه مجبورند که این مطالب رو یاد بگیرند . همچنین سعی می کنم که مطالب پایه رو سریع بگم و به جاهای جالب برسم .
در بین آموزش هم از مثال استفاده می کنم و برای هر بخش نیز تمریناتی را نیز می گذارم که اگر کسی از اول آموزش من را همراهی کند می تواند اکثر سوالات را حل کند . همچنین ممکن است جواب تمام سوالات را خودم نگذارم و تنها جواب تمریناتی را که فکر کنم مفیدتر هستند و همچنین تمریناتی که دیگران درخواست کنند را بدهم .
مشکل اصلی که الان با آن روبه رو هستم این است که باید برای این تاپیک یک فهرست تهیه کنم که خودم وقت نمی کنم این کار رو انجام بدهم و نمی دونم باید چیکار کنم .
فعلا نکته ی دیگری به ذهنم نمی رسه . اگه بعدا نکته ای به خاطرم رسید می گم .
پس فعلا .

من هستم،اساسی...
آخه هنوز تصمیم نگرفتم که ریاضی بخونم یا کامپیوتر...
چون زیاد با سوالای کامپیوتر آشنایی ندارم....
اگر هم بخوای می تونم یه 20 تایی سوال کامپیوتر(بدون حل)بذارم...
ممنون

m_honarmand_j
09-09-2007, 16:45
ترکیبیات 2-2

اصل اساسی تعمیم یافته ی شمارش : اگر r آزمایش که قرار است انجام شوند طوری باشند که اولی بتواند به هر یک از n1 برآمد ممکن منجر شود ، و اگر برای هر یک از برآمد های n1 ، n2 برآمد ممکن از آزمایش دوم وجود داشته باشد ، و برای هریک از برآمد های ممکن این دو آزمایش n3 برآمد ممکن از آزمایش شوم وجود داشته باشد ، و الی آخر ... آنگاه n1*n2*…*nr برآمد ممکن از r آزمایش وجود دارد .


مثال2-2-1 : کمیته ی طرح ریزی دانشکده ای مرکب از 3 دانشجوی سال اول ، 4 دانشجوی سال دوم ، 5 نفر دانشجوی سال سوم و 2 دانشجوی سال چهارم است . می خواهیم زیر کمیته ای که در آن 4 نفر و از هر کلاسی یک نفر شرکت دارند تشکیل دهیم . چند زیر کمیته ی مختلف می توان تشکیل داد ؟
حل: انتخاب یک زیر کمیته را می توان به عنوان ترکیب برآمد چهار آزمایش جداگانه ی انتخاب یک نماینده از هر کی از کلاسها در نظر گرفت . پس با درنظر گرفتن حالت تعمیم یافته ی اصل اساسی نتیجه می شود که 120 = 2 * 5 * 4 * 3 زیر کمیته ی ممکن وجود دارد .

مثال2-2-2: چند پلاک نمره اتومبیل مختلف می توان ساخت ، در صورتی که بدانیم از
7 جای در نظر گرفته شده ، سه جای اول با حروف و چهار جای بعدی با اعداد پر می شوند .
حل : بنا به شکل تعمیم یافته ی اصل اساسی پاسخ عبارت است از
175760000 = 10*10*10*10*26*26*26

m_honarmand_j
09-09-2007, 16:48
تمرین 2-2-1: صورت تعمیم یافته ی اصل اساسی ، شمارش را ثابت کنید .

تمرین 2-2-2: چند عدد سه رقمی وجود دارد ؟

تمرین 2-2-3: چند عدد سه رقمی فرد و چند عدد سه رقمی زوج وجود دارد ؟

تمرین 2-2-4: چند عدد چهار رقمی فرد بدون تکرار ارقام وجود دارد ؟

تمرین 2-2-5: چند عدد چهار رقمی زوج بدون تکرار ارقام وجود دارد ؟

pp8khat
09-09-2007, 17:03
آآموزش ترکیبیات 1-1
اصل جمع : فرض کنید k یک عدد طبیعی و A1 و A2 و ... و Ak ، k مجموعه ی متناهی و دو به دو مجزا باشند ؛ یعنی به ازای هر i , j = 1,2,3,…,k ، i≠j ، داشته باشیم Ai∩Aj=Ǿ . آنگاه داریم :
|Úi=1,..k Ai| =|A1ÚA2Ú…ÚAk| = ∑ki=1|Ai|
این اصل جمع به صورت مجموعه ای بود . به طور ساده تر می توان گفت که اگر برای انجام کاری k انتخب مختلف داشته باشیم A1) و A2 و ... و (Ak به طوری که هیچ کدام از آن ها با دیگری اشتراک نداشته باشند (Ai∩Aj=Ǿ) آنگاه برای آنکه بدانیم در مجموع برای انجام آن کار چند حالت مختلف داریم باید تعداد حالت های موجود برای هر انتخاب را با یکدیگر جمع بکنیم .
مثال 1-1-1: فرض کنید می خواهیم از شهر A به شهر B برویم . برای این کار می توانید از راه هوایی ، دریایی ، زمینی و ریلی استفاده کنیم . پس A1 راه هوایی ، A2 راه دریایی ، A3 راه زمینی و A4 راه ریلی می باشد . فرض کنید برای راه هوایی 2 مسیر مختلف داشته باشیم . برای راه دریایی 1 مسیر . برای راه زمینی 4 مسیر و برای راه ریلی 2 مسیر مختلف وجود داشته باشد .
یعنی : |A1| = 2 , |A2| = 1 , |A3| = 4 , |A4| = 2
می خواهیم بدانیم که به چند طریق مختلف می توانیم خود را از شهر A به شهر B برسانیم ؟
به دلیل آنکه حالت های ما (هوایی ، دریایی ، زمینی و ریلی) از هم مجزا هستند و با هم اشتراکی ندارند پس باید از اصل جمع استفاده کنیم . در نتیجه ما می توانیم به 2+1+4+2 = 9 طریق کار مورد نظر را انجام دهیم .

تمرین 1-1-1: علی 2 پیراهن آبی ، 3 پیراهن سبز ، 1 پیراهن سفید و 3 پیراهن سیاه دارد . او به چند طریق می تواند یک پیراهن برای پوشیدن انتخاب کند ؟

تو تمرین 1-1-1 شما جواب 9 میشه یا 4...
آخه اگه پیراهنا مدلشون یکسان باشن تکراری هم وجود داره...
اگه متوجه نشدین سوالمو واضح تر بگم؟

m_honarmand_j
10-09-2007, 00:39
دوست عزیز pp8khat
ممنون از تذکرتون . منظور مثلا دو نوع پیراهن آبی و ... بوده که بعد از تذکر شما اونو درست کردم .

m_honarmand_j
10-09-2007, 01:11
از دوست خوبم pp8khat ممنونم که اعلام آمادگی کردند .
( البته اگر ایشون بگند که چند سالشونه شاید بتونم یکمی راهنماییشون بکنم که المپیاد کامپیوتر چه جوریه و حتی در مورد ریاضی هم اطلاعاتی دارم . درسته تو المپیاد در نیومدم ولی خداییش الانم اطلاعاتم از بچه هایی که قبول شدند بیشتر نباشه کمتر هم نیست)
واقعا اگر کسی باشه که بتونه یک فهرست برای این تاپیک درست کنه خیلی خوب می شه . چون دیگه مجبور نمی شم سوالات و تکرار کنم و همچنین سوالهاییی که بدون جواب موندن معلوم می شن و اونا هم حل می شن و کلی فایده ی دیگر .
همچنین منتظر نظرات در مورد مطالب آموزشی هستم .
فعلا تا همینجاشو بخونید تا ادامه ی آموزش و هم بزارم .

m_honarmand_j
10-09-2007, 15:50
ترکیبیات 2-3


جایگشت ها : فرض کنید n شئ داریم .می خواهیم بدانیم که برای این n شئ چند آرایش (چینش) مختلف خطی داریم که به این تعداد آرایش مختلف جایگشت می گوییم . برای این n شئ!n(n-1)(n-2)…*3*2*1=n (بخوانید n فاکتوریل) جایگشت مختلف داریم .
زیرا در نخستین جایگاه در چینش ، هر یک از n شئ می توانند قرار بگیرند . در جایگاه دوم n-1 شئ می توانند قرار بگیرند ( زیرا یکی از اشیا در جایگاه اول قرار گرفته است) و ... و در جایگاه r ام تنها n-r+1 شئ می توانند قرار بگیرند (زیرا r-1 شئ در r-1 جایگاه اول قرار گرفته اند) . پس طبق اصل اساسی شمارش !n حالت مختلف برای چینش n شئ وجود دارد .


مثال 2-3-1: چند ترتیب مختلف ( جایگشت) برای یک تیم بسکتبال مرکب از 9 بازیکن وجود دارد ؟
حل : تعداد ترتیبهای ممکن 362880 = !9 است .
مثال 2-3-2: کلاسی مرکب از 6 دانشجوی پسر و 4 دانشجوی دختر است . بعد از امتحان ، دانشجویان بر اساس نمراتشان رتبه بندی می شوند . فرض کنید هیچ دو دانشجویی نمره ی یکسانی نگرفته باشند .
(الف) چند رتبه بندی مختلف مجود دارد؟
(ب) اگر دو گروه دانشجو بین خودشان رتبه بندی شوند ، چند رتبه بندی مختلف مجود دارد ؟

حل : (الف) چون هر رتبه بندی متناظر با آرایش مرتب شده ی خاصی از 10 نفر است ، پس پاسخ این قسمت عبارت است از 3628800 = !10 .
(ب) چون !6 رتبه بندی ممکن از پسران دانشجو و !4 رتبه بندی ممکن از دختران دانشجو بین خودشان وجود دارد از اصل اساسی نتیجه می شود که 1728 = (24)(720) = (!4)(!6) رتبه بندی در این حالت وجود دارد .

m_honarmand_j
10-09-2007, 15:57
تمرین 2-3-1: چند پلاک مختلف ماشین با 7 علامت ممکن است اگر دو علامت اول از حروف و 5 علامت دیگر از اعداد ترکیب شده باشند .
با این فرض که هیچ حرف یا عددی در یک پلاک نمی تواند تکرار شود قسمت قبل را تکرار کنید .

تمرین 2-3-2: یک گروه نوازنده چهار نفری ، با 4 آلت موسیقی باهم همکاری دارند . اگر هر یک از این افراد چهار آلت موسیقی را بتوانند بنوازند ، چند آرایش مختلف ممکن است ؟ اگر دونفر آنها هر چهار آلت موسیقی ولی دونفر دیگر هر یک تنها پیانو و طبل بنوازند ، چند آرایش مختلف امکان دارد؟

تمرین 2-3-3: اگر 4 امریکایی ، 3 فرانسوی و 3 انگلیسی بخواهند در یک ردیف بنشینند ، در حالی که افراد با ملیت یکسان قرار باشد کنار هم بنشینند ، به چند طریق می توانند این کار را انجام دهند ؟

m_honarmand_j
10-09-2007, 16:10
سلام
دوستان همونطور که قول داده بودم (البته قولی هم نداده بودم) دارم به سرعت مطالب پایه رو می ذارم و البته به جاهای جالبش رسیدیم که از این به بعد سوالات جالب تری می شه طرح کرد .
همچنین سعی می کنم که جواب تمرینات اولیه رو بذارم که اونایی که تازه کارن نحوه ی حل اینجور مسائل و یاد بگیرن و اوناییهم که بلدن یک کم به حرکت واداشته بشن و از این به بعد اونا جوابارو بگن .

البته اگه دیدید توی مطالب اشکال تایپی وجود داره و یا اینکه چیزی اشتباهه حتما بهم بگید .
همچنین یک کم توی این فرمول ها به مشکل برخوردم چون من قبلا توی word مطالب و می نویسم و بعد کپی می کنم اینجا یک کم قاطی می شه .

از pp8khat هم می خوام اون سوال هایی رو که گفته بذاره . (اون 20 تا سوالی که گفتی داری)

منتظر انتقادات و پیشنهادات در مورد مطالب آموزشی هم هستم .

pp8khat
12-09-2007, 16:22
سلام.
طبق قولی که داده بودم سوالاتمو گذاشتم البته 60 تاست(پدرم در اومد تا تایپشون کردم)...
به جز سوال های دنباله دار فعلاً تو هر هر پست 2 تا سوال میذارم(اگه سختتونه که بخونید بگید تو هر پست یک سوال بذارم یا برعکس بیشتر بذارم)
چون تایپ همه ی سوالا زمان می بره اون ها رو تیکه تیکه کردم...
تو سری اول 14 تا می ذارم...

X1-در جریان یک مبارزه انتخاباتی محلی،هشت نامزد از حزب A و پنج نامزد از حزب B برای اشغال پست ریاست هیات
مدیره مدرسه معرفی شده اند.
1-اگر بنا باشد یکی از این نامزد ها رئیس بشود،چند امکان برای برنده احتمالی وجود دارد؟
الف)15
ب)40
ج)13
د)45

2- چند امکان وجود دارد که 2تا از نامزدها(یک نفر از هر حزب)برای اشغال پست بالا مقابل هم قرار گیرند؟
الف)15
ب)40
ج)13
د)45

pp8khat
12-09-2007, 16:23
X2-اتومبیل های بیوک در 4 مدل،12 رنگ،3 اندازه متفاوت برای موتور و 2 نوع جعبه دنده به بازار می آیند.
3-چند بیوک متمایز می توان ساخت؟
الف)288
ب)24
ج)258
د)42

4-اگر یکی از رنگ هایی که در دسترس است آبی باشد،چند بیوک آبی متفاوت می توان ساخت؟
الف)288
ب)24
ج)258
د)42

pp8khat
12-09-2007, 16:24
5-دو خانم جوان در حالی که مشغول خرید بودند،دیدند که دو مرد،درست قبل از آنکه دزدگیر به صدا درآید،با اتومبیل از جلوی یک مغازه جواهر فروشی دور شدند.گرچه همه چیز سریع اتفاق افتاد،ولی هنگامی که این دو خانم مورد بازجویی قرار گرفتند،توانستند اطلاعات زیر را درباره شماره اتومبیل(که مرکب بود از دو حرف و سپس چهار رقم)به پلیس بدهند.
یکی از آن دو یقین داشت که حرف دوم شماره اتومبیل Oیا Q بوده و آخرین رقم 3 یا 8 بود.
دیگری به بازجو گفت که حرف اول شماره اتومبیل Cیا G و رقم نخست قطعاً 7 بود.
پلیس باید چند شماره اتومبیل مختلف را بررسی کند؟
الف)800
ب)600
ج)850
د) 650


6- یک اغذیه فروشی برای تبلیغات اعلام کرده است که هر مشتری می تواند،در صورت تمایل،همبرگر خود را با بعضی از مواد زیر یا همه ی آن ها یا بدون هیچ یک از آن ها بخورد:
سس گوجه فرنگی،خردل،مایونز،کاهو،گو جه فرنگی،پیاز،خیارشور،پنیر یا قارچ.
چند نوع متفاوت همبرگر می توان سفارش داد؟
الف)1024
ب)256
ج)36
د)512

pp8khat
12-09-2007, 16:24
X3-در یک فروشگاه هشت نوع مختلف نان روغنی و شش نوع مختلف کلوچه فروخته می شود.علاوه بر این اقلام می توان از نوشیدنی های زیر در اندازه های کوچک،متوسط یا بزرگ نیز خریداری کرد:
قهوه(سیاه با کرم،با شکر،با کرم و شکر)،چای(معمولی،با کرم،با شکر،با کرم و شکر،با لیمو ترش یا با لیمو ترش وشکر)،کاکائوی داغ و آب پرتغال.
مشتری این فروشگاه به چند طرق می تواند:
7-یک نوع نان ویک نوشیدنی سفارش دهد؟
الف)168
ب)861
ج)681
د) 18144

8-یک نوع نان و یک قهوه برای خودش و یک کلوچه و یک چای برای دوستش سفارش دهد؟
الف)168
ب)861
ج)7315
د)18144

9-یک قطعه نان روغنی و یک چای برای خودش،یک کلوچه و یک آب پرتغال برای دوستش و یک نوع نان و یک قهوه
برای هر یک از دو دستیارش سفارش دهد؟
الف)731168
ب)156608731
ج)73156608
د)18144

pp8khat
12-09-2007, 16:25
X4-سه شهر کوچک که با A,Bو C نشان داده شده اند،به وسیله ی مجموعه ای از جاده های دوطرفه،همان طوری که در شکل نشان داده شده است،به هم مرتبط شده اند.
[ برای مشاهده لینک ، لطفا با نام کاربری خود وارد شوید یا ثبت نام کنید ]
10- به چند طریق می توان از شهر A به شهر C رفت؟
الف)14
ب)182
ج)41
د)196

11-به چند طریق می توان سفر رفت و برگشت از شهر A به شهر C ترتیب داد؟
الف)14
ب)182
ج)41
د)196

12-چند تا از سفر های رفت و برگشتی سوال 11 به گونه ای هستند که سفر بازگشت(از شهر C به شهر A) حداقل به طور جزئی،متفاوت با مسیری است که در سفر از شهر A به شهر C در پیش گرفته شده است؟(به عنوان مثال،اگر سفر رفت در طول جاده های R1 و R6 از شهر A به شهر C صورت گرفته باشد،آنگاه در بازگشت می توان،بین امکانات گوناگون جاده های R6 و R3 یا جاده های R5 و R1 یا جاده های R7 و R2 یا جاده R9 را در پیش گرفت،ولی جاده های R6 و R1 انتخاب نخواهد شد)
الف)14
ب)182
ج)190
د)196

pp8khat
12-09-2007, 16:27
13-به چند طریق می توان نماد های a,b,c,d,e,e,e,e,e را مرتب کرد به طوری که هیچ eای کنار e دیگری قرار نگیرد؟
الف)18
ب)16
ج)42
د)24

14-
ببخشید که اینقدر داغون شده...
[ برای مشاهده لینک ، لطفا با نام کاربری خود وارد شوید یا ثبت نام کنید ]

m_honarmand_j
13-09-2007, 14:43
با تشکر از دوست خوبم pp8khat که سوالات و قرار دادند .
من خیلی خوشحال شدم وقتی دیدم که سوالات با مطالب آموزشی هماهنگ هستند و مربوط به آنها می شوند .

اولا امیدوارم که این همکاری pp8khat ادامه پیدا کنه و بعدش امیدوارم که سوالاتی که قرار داده می شه با مطالب آموزشی هماهنگ باشند و در آخر هم امیدوارم که خودم مجبور نشم که جواب همه ی این سولات و بزارم .


(دوستان هنوز منتظر یک نفر هستم تا برای ایجاد فهرست برای این تاپیک اعلام آمادگی کنه)

m_honarmand_j
13-09-2007, 14:50
دوستان من در تایپ فرمول ها مشکل پیدا کردم . کسی پیشنهادی نداره؟؟؟؟

eh_mn
13-09-2007, 16:52
دوستان من در تایپ فرمول ها مشکل پیدا کردم . کسی پیشنهادی نداره؟؟؟؟
با سلام.
به اتاق Tex سر زدين؟
فهرست مطالب انجمن رو هم ببينيد!
در مورد فرمول‌نويسي و انتشار اون در وب اطلاعات كاملي گفته شده.

منظورتون چه جور فهرستيه؟ (از نظر دسته بندي مطالب بطور موضوعي و...)
اگر بخوايم براي سوالات فهرست درست كنيم چطور به يك سوال ارجاع بديم؟ (مثلا با شماره به سوالات ارجاع بديم يا با متن سوال(!!!) )
موفق باشيد.

m_honarmand_j
14-09-2007, 12:17
سلام

ممنون به اتاق tex سر زدم . ولی فکر کنم کمی طول بکشه تا اون فرمول مول نویسی و یاد بگیرم و همچنین یک کم باید زمان بیشتری و صرف کنم تا بتونم فرمول ها رو آماده کنم . پس اگه مطالب دیر شد امیدوارم که دلخور نشید .


فهرست موضوعی می خواهم . اگه بخواهیم برای سوالات فهرست درست کنیم خوب مثلا یک دسته درست می کنیم و اسمشو می ذاریم اصل لانه کبوتری و تمام مطالب و سوالات در مورد اصل لانه کبوتری و لینکشون (که در این تاپیک هستند)و قرار می دیم اونجا . ( می بینید بالا سمت چپ هر پست یک شماره هست که اگر روش کلیک کنیم صفحه ای می آره که فقط اون پست و نشون می ده ، می خوام اینجوری پست های مفید دسته بندی بشن و لینکشون در یک فهرت قرار داده بشه)
نمی دونم تونستم منظورم و برسونم یا نه؟

pp8khat
14-09-2007, 15:43
سلام

ممنون به اتاق tex سر زدم . ولی فکر کنم کمی طول بکشه تا اون فرمول مول نویسی و یاد بگیرم و همچنین یک کم باید زمان بیشتری و صرف کنم تا بتونم فرمول ها رو آماده کنم . پس اگه مطالب دیر شد امیدوارم که دلخور نشید .


فهرست موضوعی می خواهم . اگه بخواهیم برای سوالات فهرست درست کنیم خوب مثلا یک دسته درست می کنیم و اسمشو می ذاریم اصل لانه کبوتری و تمام مطالب و سوالات در مورد اصل لانه کبوتری و لینکشون (که در این تاپیک هستند)و قرار می دیم اونجا . ( می بینید بالا سمت چپ هر پست یک شماره هست که اگر روش کلیک کنیم صفحه ای می آره که فقط اون پست و نشون می ده ، می خوام اینجوری پست های مفید دسته بندی بشن و لینکشون در یک فهرت قرار داده بشه)
نمی دونم تونستم منظورم و برسونم یا نه؟


دوستان من در تایپ فرمول ها مشکل پیدا کردم . کسی پیشنهادی نداره؟؟؟؟

می تونید از MathType هم استفاده کنید که خیلی به درد بخوره و آسون...:27::40::8:
لینک دانلود:

برای مشاهده محتوا ، لطفا وارد شوید یا ثبت نام کنید
این سریال مال ورژن 5.2c بود(که من دارم ازش استفاده می کنم)حالا شاید رو ورژن 6 هم کار کنه...:13:
MTWE521-011740-7SDWR
این هم دو تا سایت برای آپلود تصاویر گزاره های ریاضی و فرمول هات::20:

برای مشاهده محتوا ، لطفا وارد شوید یا ثبت نام کنید
ضمناً میشه جواب مثالهای ترکیبات 1-1 تون رو بذارید(چند عدد وجود دارد ها...)؟
راستی اگه منم بعضی از سوال هایی رو که گذاشتم اونایی رو که بلدم رو حل کنم،اشکال نداره؟:43::31:
یه سوال دیگه هم داشتم؛ آیا المپیاد ریاضی و کامپیوتر در یک زمان و در یک روز برگزار می شود؟(سال اول که این شکلی بود:41:)

pp8khat
14-09-2007, 18:14
X5-یک استاد علوم کامپیوتر هفت کتاب برنامه نویسی متفاوت در قفسه ای از کتابخانه دارد.سه تا از این

کتاب ها درباره فورتارن است.چهار تای دیگر درباره بیسیک است.این استاد به چند طریق می تواند این

کتاب ها را در قفسه مرتب کند؟
15-در صورتی که هیچ قیدی در کار نباشد؟
الف)4050
ب)5040
ج)5400
د)4500
16-در صورتی که بنا باشد کتاب های مربوط به زبان های یاد شده یک در میان قرار گیرند؟
الف) 144
ب)241
ج)288
د)92
17-در صورتی که همه ی کتاب های فورترن کنار هم قرار گیرند؟
الف)288
ب)380
ج)512
د)720
18-در صورتی که همه ی کتاب های فورترن کنار یکدیگر و همه ی کتاب های بیسیک نیز کنار یکدیگر

قرار گیرند؟
الف)288
ب)380
ج)512
د)720

pp8khat
14-09-2007, 18:15
21-تعداد مسیر های(پلکانی)واقع در صفحه xy از(1و2)به(4و7) را تعیین کنید که هر مسیر از پله هایی
تشکیل شده است که یک واحد به راست(R)یا یک واحد به بالا(U) می رود.
الف)65
ب)56
ج)256
د)536
22-چند مسیر متمایز سه بعدی اقلیدیسی از (0و2و1-) به (7و3و1) وجود دارد،در صورتی که هر حرکت
یکی از انواع زیر باشد؟
(A):(x,y,z)==>(x,y,z+1)
(V):(x,y,z)==>(x,y+1,z)
(H):(x,y,z)==>(x+1,y,z)
الف)100
ب)56
ج)360
د)536
23- چند تا از این نوع مسیرها با توجه به سوال22 از(5و0و1) به (7و1و8)وجود دارد؟
الف)100
ب)56
ج)360
د)536

pp8khat
14-09-2007, 18:17
-حداکثر چند زیر مجموعه از مجموعه x={1,2,3,4,5}a (آ بیخوده برای تایپ بهتر)می توان انتخاب کرد،به طوری که اجتماع هیچ دو زیر مجموعه انتخاب شده مساوی با مجموعه x نشود؟
الف)16
ب)10
ج)26
د)20
27-حداکثر چند تا از دایره های شکل زیر را می توان پر کرد به طوری که هیچ چهار دایره پر شده ای
رئوس یک مربع یا مستطیل با اضلاع افقی و عمودی نباشند؟
الف)6
ب)10
ج)9
د)8
[ برای مشاهده لینک ، لطفا با نام کاربری خود وارد شوید یا ثبت نام کنید ]
28-با ارقام 3و5و7 به چند طریق می توان یک عدد چهار رقمی ساخت که بر 3 بخش پذیر باشد؟
(تکرار ارقام مجاز است)
الف)27
ب)21
ج)26
د)19

pp8khat
14-09-2007, 18:18
- به چند طریق می توان حروف واژه VISITING را مرتب کرد؟
الف)6720
ب)380
ج)512
د)720
20-در چند تا از ترتیب های سوال 19 هر سه I کنار هم قرار دارند؟
الف)6720
ب)380
ج)512
د)720

pp8khat
14-09-2007, 18:19
24- قطعه برنامه پاسکال زیر را،که در آن i,j,k متغیرهایی صحیح اند درنظر می گیریم.دستور Writeln
چند بار اجرا می شود؟
For i:=1 to 12 do
For j:=5 to 10 do
For j:=15 downto 8 do
Writeln((i-j)*k)c
c هیچی نیست برای تایپ بهتر گذاشتمش...
25-
[ برای مشاهده لینک ، لطفا با نام کاربری خود وارد شوید یا ثبت نام کنید ]

m_honarmand_j
15-09-2007, 12:53
می تونید از MathType هم استفاده کنید که خیلی به درد بخوره و آسون...:27::40::8:
لینک دانلود:

برای مشاهده محتوا ، لطفا وارد شوید یا ثبت نام کنید
این سریال مال ورژن 5.2c بود(که من دارم ازش استفاده می کنم)حالا شاید رو ورژن 6 هم کار کنه...:13:
MTWE521-011740-7SDWR
این هم دو تا سایت برای آپلود تصاویر گزاره های ریاضی و فرمول هات::20:

برای مشاهده محتوا ، لطفا وارد شوید یا ثبت نام کنید
ضمناً میشه جواب مثالهای ترکیبات 1-1 تون رو بذارید(چند عدد وجود دارد ها...)؟
راستی اگه منم بعضی از سوال هایی رو که گذاشتم اونایی رو که بلدم رو حل کنم،اشکال نداره؟:43::31:
یه سوال دیگه هم داشتم؛ آیا المپیاد ریاضی و کامپیوتر در یک زمان و در یک روز برگزار می شود؟(سال اول که این شکلی بود:41:)



سلام

ممنون از راهنماییتون . (ولی بابا 5mb حجمش . کی حوصله داره بشینه دانلودش کنه . تازه اگه حوصله هم داشته باشم ، سرعتم نمی کشه)
چشم جواب هارو می ذارم .
هر کدوم و که بلدید و دیدی بعد از مدتی کسی جوابی بهش نداد رو خودتون جابشو بذارید .
المپیاد ریاضی و کامپیوتر همزمان برگزار نمی شن . (مرحله ی اولشون حداقل یک هفته فاصله دارن و مرحله ی دومشون هم دوهفته با هم فاصله دارن)

m_honarmand_j
21-09-2007, 02:08
تمرین 1-1-1: علی 2 نوع پیراهن آبی ، 3 نوع پیراهن سبز ، 1 نوع پیراهن سفید و 3 نوع پیراهن سیاه دارد . او به چند طریق می تواند یک پیراهن برای پوشیدن انتخاب کند ؟


جواب تمرین 1-1-1 : بنا به اصل جمع او به 2+3+1+3 = 9 طریق می تواند یک پیراهن برای پوشیدن انتخاب کند .

m_honarmand_j
21-09-2007, 02:13
تمرین 1-2-1: علی 5 نوع شلوار ، 10 نوع پیراهن ، 8 نوع کراوات و 2 نوع ساعت دارد . او به چند طریق می تواند لباس بپوشد ؟


جواب تمرین 1-2-1 : بنا به اصل ضرب او به 5*10*8*2 = 800 طریق می تواند لباس بپوشد .

m_honarmand_j
21-09-2007, 02:17
تمرین 2-1-1: چند عدد یک رقمی و چند عدد دو رقمی وجود دارد ؟


جواب تمرین 2-1-1 : در جای یکان هر یک از از 10 رقم موجود ( 0، 1 ... ، 9) می توانند قرار گیرند پس ما 10 عدد (اگر صفر را هم به حساب بیاوریم) یک رقمی داریم .
در جایگاه دهگان هر یک از اعداد 1 تا 9 می توانند قرار بگیرند و در جایگاه یکان هر یک از 10 عدد صفر تا 9 می توانند قرار بگیرند ، پس بنا به اصل ضرب ما 9*10 = 90 عدد دورقمی داریم .

m_honarmand_j
21-09-2007, 02:19
تمرین 2-1-2: چند عدد دو رقمی بدون تکرار ارقام وجود دارد ؟


جواب تمرین 2-1-2 : در جایگاه دهگان هر یک از اعداد 1 تا 9 می توانند قرار بگیرند و در جایگاه یکان هر یک از اعداد صفر تا 9 به جز عددی که در جایگاه دهگان قرار گرفته است می توانند قرار بگیرند پس روی هم 9 عدد در جایگاه دهگان و 9 عدد در جایگاه یکان می توانند قرار بگیرند و بنا به اصل ضرب روی هم می شود 9*9=81 عدد دورقمی بدون تکرار ارقام .

m_honarmand_j
21-09-2007, 02:22
تمرین 2-1-3: چند عدد دو رقمی فرد وجود دارد ؟


جواب تمرین 2-1-3: در جایگاه یکان باید یکی از اعداد 1،3،5،7یا9 را قرار بدهیم . پس برای این جایگاه 5 انتخاب و در جایگاه دهگان هر یک از اعداد 1 تا 9 می توانند قرار بگیرند . پس برای جایگاه دهگان 9 انتخاب داریم و در نتیجه بنا به اصل ضرب 9*5=45 عدد دورقمی فرد خواهیم داشت .

m_honarmand_j
21-09-2007, 02:25
تمرین 2-1-4: چند عدد دو رقمی زوج وجود دارد ؟


جواب تمرین 2-1-4: برای آنکه عددی زوج باشد باید در جایگاه یکان آن یکی از اعداد 0،2،4،6یا 8 قرار بگیرند . فرض کنید رقم 0 در جایگاه یکان قرار بگیرد . آنگاه برای جایگاه دهگان هر یک از اعداد 1 تا 9 می توانند قرار بگیرند در نتیجه 9*1=9 عدد زوج بوجود می آید . حال فرض کنید یکی از اعداد 2یا 4 یا 6 یا 8 در جایگاه یکان قرار بگیرند . آنگاه در جایگاه دهگان هریک از اعداد 1 تا 9 می توانند قرار بگیرند یعنی 9 انتخاب و در نتیجه 4*9=36 عدد زوج بوجود می آید و همچنین از قبل 9 عدد زوج نیز داریم پس بنا به اصل جمع روی هم 36+9=45 عدد دورقمی زوج خواهیم داشت . (توجه کنید که چون صفر در جایگاه دهگان نمی تواند قرار بگیرد ما مجبور بودیم که جواب را در دو حالت مجزا برسی کنیم و مانند تمرین ( 2-1-3) عمل نکردیم)
راه حل دوم : اعداد دورقمی به دو دسته روز و فرد تقسیم می شوند . اگر عددی زوج نباشد پس فرد است . حال اگر مجموعه ی اعداد زوج دورقمی را A در نظر بگیریم و مجموعه ی مرجع M را اعداد دورقمی ، آنگاه A' برابر اعداد فرد دورقمی می شود . حال چون در تمرین قبل تعداد اعداد دورقمی فرد را بدست آورده ایم می توانیم بنا به اصل متمم تعداد اعداد دورقمی زوج را بدست بیاوریم که تعداد اعداد دورقمی 90 تا است (تمرین 2-1-1) و تعداد اعداد دورقمی فرد 45(تمرین 2-1-3) پس تعداد اعداد دورقمی زوج برابر 45 می شود .

m_honarmand_j
21-09-2007, 02:30
تمرین 2-1-5: چند عدد دو رقمی فرد بدون تکرار ارقام وجود دارد ؟


جواب تمرین 2-1-5: مانند تمرین( 2-1-2) عمل کنید . (جواب برابر 40 است)

m_honarmand_j
21-09-2007, 02:39
تمرین 2-1-6: چند عدد دورقمی زوج بدون تکرار ارقام وجود دارد ؟


جواب تمرین 2-1-6: مانند تمرین ( 2-1-2) و تمرین (2-1-4) عمل کنید . (جواب برابر 41 است)

m_honarmand_j
21-09-2007, 03:04
سلام بر دوستان
تا اینجاشو داشته باشید تا بعد ادامشو هم بزارم .

m_honarmand_j
21-09-2007, 23:29
سلام
دوستان با فرا رسیدن مهر ماه فکر کنم نتونم تند تند مطالب جدید بزارم .

دوستان من هنوز منتظر اعلام آمادگی برای ایجاد فهرست هستم .

اگر کسی در میان مطالب آموزشی و کل در بین پست ها اشتباهی دید لطفا اطلاع بده .

m_honarmand_j
23-09-2007, 18:02
دوستان یک کم سرم شلوغ ه . به زودی ادامه ی مطالب و هم می زارم.

m_honarmand_j
06-10-2007, 01:06
دوستان مشکل فرمول ها رو حل کردم . به زودی ادامه ی آموزش رو هم می ذارم . همچنین می بینید که بعد از اینکه ادامه ی آموزش و گذاشتم ، کم کم مسائلی که pp8khat گذاشتن به راحتی حل می شوند . همچنین سئ می کنم تا جواب اون سوالات رو هم بذارم اما اگه دلیل جواب هارو نفهمیدید با خواندن ادامه ی مطالب متوجه خواهید شد ، پس نگران نباشید .

robert
12-11-2007, 22:57
سلام بر یاران همیشگی پی سی ورلد و خصوصا ریاضیدانان بزرگوار !!!
مدتی هست که برای حل دستگاه های معادلات سه معادله سه مجهول و چهار معادله چهار مجهول درس مدار الکتریکی دانشگاه دنبال روش حل چنین دستگاه هایی به روش کرامر هستم !!! منتها چون تعالیم دبیرستان رو تا حدودی فراموش کردم لذا از شما بزرگواران خواهشمندم یه لینکی چیزی معرفی کنید تا بلکه این روش حل دستگاه معادلات یادم بیاد !!!
ممنونم !!!

behroozifar_s
14-11-2007, 21:44
شاید این برنامه کمک کنه :


#include <iostream.h>
#include <conio.h>
int calc(int [],intdim);
void revmatrix( int [],intdim);
voidmain()
{
intmatrix[1000];
intdim,temp;
doubleleftsum,rightsum;
cout<<" PLEASE ENTER MATRIX DIMENSION : ";
cin>>dim;
cout<<"\n\n\n";
for( int i = 0;i<( dim*dim );i ++ )
{
cout<<"ENTER ELEMAN : ";
cin>>temp;
matrix[i] = temp;
clrscr();
cout<<" PLEASE ENTER MATRIX DIMANTION : "<<dim;
cout<<"\n\n\n";
}//for iif ( dim > 2 )
{
leftsum = calc( matrix , dim );
cout<<"LEFTSUM of the matrix = "<< leftsum <<"\n\n";
revmatrix( matrix , dim );
rightsum = calc( matrix ,dim );
cout<<"RIGHTSUM of the matrix = "<< rightsum <<"\n\n\n\n\n\n" ;
cout<<" ( DETERMINAN OF THE MATRIX = "<< leftsum - rightsum<<" )";
}

elsecout<<" ( DETERMINANOF THE MATRIX = "<<(matrix[0] * matrix[3] - matrix[1] * matrix[2])<<" )";

getch();
}//endmain

/////////////////calc function//////////

int calc( intmatrix[ ], intdim )
{
int sum = 0, bul, x = 1;
for( int l = 0; l<( dim*dim );l += ( dim+1 ) )//ghotraslix *= matrix[l];
sum = x;
x = 1;
for( int c = 1;c<dim;c ++ )
{
bul=c;
for( int m = 0;m<dim;m ++ )
{
if( ( bul+1 )%dim != 0 )
{
x *= matrix[bul];
bul +=( dim + 1 );
}

else
{
x = x * matrix[bul];
bul += 1;
}

}//for msum += x;
x = 1;
}//for c

البته گفتن این نکته خالی از لطف نیست که :

a11A11 + a21A21 + ...............+ an1An1 = det A
,

Ri= Aji , Ai2 , ......... Aij , ...........Ain


که :

ri . Ri = det A
ri . Rj = 0 , j <>i

یا بهتر بگم :

ri .Ri = q ij . det A
( qij همان دلتای کرونکر است که چون اینجا دلتا نداشت این طوری نشون دادم و ri سطر i ام ماتریس A است . )

توجه کنید که : b1=a1nxn + ....... + a12x1 + a11x1 و همین طور تا آخر .

پس از بسط رابطه بالا داریم :





| b1 a12 ….……….a1n|
1 |b2 a22 ….…..….. a2n |
X1 = ------- | ……………….........……|
Det A |bn an2 … … ann|


|


برای بقیه xi ها هم همین رابطه را استفاده می کنیم .



|b1 a12 ……...…….a1n|
|b2 a22 …..….….. a2n |
|…………………........………|
|bn an2 …...............ann|
X1 = -------------------------------
|a11 a12 ……...…….a1n|
|a12 a22 …..….….. a2n |
|……………………….........…|
|an1 an2..................ann|

ان شاء الله شکل دترمینان و ماتریس رو در فرمولهای بالا تشخیص بدهید . ( سعی کردم قابل فهم باشه )




البته شما استاد هستید این مطلب هم فقط جهت یاد آوری بود . اگر خوب متوجهنشدید می تونم بازم کمک کنم.


پیروز باشید

robert
14-11-2007, 22:56
ممنون عزیز :11:!!! برنامه خوبی نوشتی !!! :20:البته بین متغیر های int فاصله ننداخته بودی که باعث شد موقع کامپایل هفده تا ارور بده که خودم درستش کردم !!!
بازهم تشکر از اینکه هم میدونستی من تو سی پلاس پلاس برنامه مینویسم و هم جواب سوالمو دادی و باعث شدی یه چیزهایی یادم بیاد !!!:46: ولی من بیشتر دنبال روش تئوری حل دستگاه معادلات خطی با استفاده از روش کرامر هستم !!! برای درس مدار یک خیلی به کار میاد !!

mir@
14-11-2007, 22:59
در اينجا بسيار زيبا توضيح داده شده

[ برای مشاهده لینک ، لطفا با نام کاربری خود وارد شوید یا ثبت نام کنید ]'s_rule

behroozifar_s
14-11-2007, 23:19
در اينجا بسيار زيبا توضيح داده شده

[ برای مشاهده لینک ، لطفا با نام کاربری خود وارد شوید یا ثبت نام کنید ]'s_rule



ممنون که شما بهتر توضیح دادید . برای من هم مفید بود .

پیروز باشید

robert
14-11-2007, 23:33
در اينجا بسيار زيبا توضيح داده شده

[ برای مشاهده لینک ، لطفا با نام کاربری خود وارد شوید یا ثبت نام کنید ]'s_rule

ممنون !! بدردم میخوره !!! هم از شما و هم از behroozifar_s خیلی خیلی ممنونم !!:11::11::11:

SuB
04-12-2007, 20:36
سلام
دوست عزیز منبه سوالات شما چی هست؟ خودتون ابداع می‌کنید یا از جایی می‌نویسید؟

V A H I D
16-12-2007, 13:50
1.حل دستگاه های دو معدله دو مجهولی غیر خطی چگونه است؟؟؟
2.دستگاه معادله ی خطی همگن چگونه است؟ تعداد جوابشان چگونه بدست می اید
3.معکوس ماتریس 3در 3 چگ.نه بدست می آید ؟ وبعد دستگاه 3 معادله 3 مجهولی به روش کرامر و به روش معکوس
چگونه بدست می اید؟؟
خواهش می کنم بگید اشکال نداره زیاد هم باشه برای پس فردا می خواهم....

pp8khat
16-12-2007, 14:25
دترمینان ماتریس های مرتبه سوم:
[ برای مشاهده لینک ، لطفا با نام کاربری خود وارد شوید یا ثبت نام کنید ]
امیدوارم آلگوریتمشو فهمیده باشی...

V A H I D
16-12-2007, 15:40
خوب ای ت دترمینانش هست خوب بقیه اش چی ؟؟؟

pp8khat
17-12-2007, 21:02
عزیز با استفاده از قانون ساروس می تونی دستگاه های n معادله n مجهولی خطی رو حل کنی...
لطفاً یکی قانون ساروس توضیح بده!من حال ندارم!
ممنون

m_honarmand_j
19-12-2007, 23:34
سلام دوست عزیز SuB
اگه بخوام منبع مشخصی و بگم نمی شه . چون منبع خاصی نیست و از کتاب ها ی مختلفی سوال ها رو انتخاب می کنم . بعضی از سوال ها رو هم جایی شنیدم و نمیدونم که منبع ثبت شده ای دارن یا نه و بعضی از سوال ها رو هم خودم طرح کردم .
اگه منبعی در موضوع خاصی می خواهید ، بگید تا معرفی کنم .
امیدوارم که موفق باشید .

no name
23-12-2007, 16:02
سلام
من می خواستم ببینم که چه جوری می شه تعداد مربعات یه مربع n*n رو محاسبه کرد
فرمولشو می خواستم از اساتید
مثلا در مربع 2*2 5 تا مربع هست
در 3*3 15 تا هست

no name
23-12-2007, 16:07
ای اساتید ریاضی کمککککککککککککک
کجایید؟؟؟؟؟؟؟؟؟؟

mir@
23-12-2007, 17:15
در یک مربع 2 در 2 ، پنج مربع وجود دارد: 1+4
در یک مربع 3 در 3 چهارده مربع وجود دارد: 1+4+9=14
در یک مربع 4 در 4 سی مربع وجود دارد: 1+4+9+16=30
.................................................. ...............
.................................................. ...............
و دریک مربع 8 در 8 دویست و چهار مربع وجود دارد: 1+4+9+16+25+36+49+64=204


برای مشاهده محتوا ، لطفا وارد شوید یا ثبت نام کنید

[ برای مشاهده لینک ، لطفا با نام کاربری خود وارد شوید یا ثبت نام کنید ]^2+\cdots+n^2=\frac{n(n+1)(2n+1)}{6 }

no name
31-12-2007, 23:07
مرسی عزیزم

hamid6885
01-01-2008, 10:09
عزیز با استفاده از قانون ساروس می تونی دستگاه های n معادله n مجهولی خطی رو حل کنی...
لطفاً یکی قانون ساروس توضیح بده!من حال ندارم!
ممنون

دکارت عزیز!تا اونجایی که من میدونم دستور ساروس برای حل دترمینان های 3*3 است و چه ربطی به n معادله n مجهول داره؟!
اگه میشه بیشتر توضیح بده:18:

pp8khat
02-01-2008, 10:51
اسمم pp8khat هستش!:46:
فکر کردم کم توضیح بدم تا خودت بری دنبالش بهتر باشه و بهتر یاد بگیری...
می دونی؛از ابتدایی بهمون یاد دادن که لقمه رو یکی دیگه بذاره تو دهنمون(البته منظورم با همه است).
از بسط قانون ساروس می تونی جواب معادله های "n معادله n مجهول خطی"رو به دست بیاری...
[ برای مشاهده لینک ، لطفا با نام کاربری خود وارد شوید یا ثبت نام کنید ]
[ برای مشاهده لینک ، لطفا با نام کاربری خود وارد شوید یا ثبت نام کنید ]
این عکس ها از کتاب "فرمول ها،آحاد و ارقام در علوم مهندسی"برداشته شده(تالیف K.Gieck و ترجمه مهندس مجید بدیعی)

pp8khat
02-01-2008, 11:01
دو تا عکس گذاشته بودم !!نمی دونم چرا یکیشون غیب شد!
اشکال نداره اون یکی رو تو این پست می ذارم...(اگه نمی تونی ببینی رو عکس راست کلیک کن و Save Image As رو انتخاب کن...)
[
برای مشاهده محتوا ، لطفا وارد شوید یا ثبت نام کنید

Antonio Andolini
07-01-2008, 16:09
دو تا عکس گذاشته بودم !!نمی دونم چرا یکیشون غیب شد!
اشکال نداره اون یکی رو تو این پست می ذارم...(اگه نمی تونی ببینی رو عکس راست کلیک کن و Save Image As رو انتخاب کن...)
[
برای مشاهده محتوا ، لطفا وارد شوید یا ثبت نام کنید


----- هست. راهی به جز گذاشتن عکس را ه حل برای نوشتن فرمول و از این چیزا این جا نیست؟

sherlockholmz
09-01-2008, 10:22
1.
2.دستگاه معادله ی خطی همگن چگونه است؟ تعداد جوابشان چگونه بدست می اید

خواهش می کنم بگید اشکال نداره زیاد هم باشه برای پس فردا می خواهم....

سلام،
به دستگاه n معادله ،n مجهول كه طرف ثابت آن تماما" صفر باشد، دستگاه معادله همگن مي گويند.
در اين نوع معادلات ،دترمينان ضرائب معين كننده نوع جواب است.
اگر دترمينان ضرائب يك دستگاه معادله همگن مساوي با صفر باشد، معادله بي نهايت جواب غير صفر داردو در غير اين صورت، بردار جواب دستگاه يك بردار صفراست.به مثالهاي زير توجه نمائيد:
[ برای مشاهده لینک ، لطفا با نام کاربری خود وارد شوید یا ثبت نام کنید ]

m_honarmand_j
10-01-2008, 16:19
سلام بر همه ی دوستان
یک مدت نبودم . البته قرار بود که مطالب آموزشی رو بزارم ولی به علت بد قولی یکی از دوستانم که قرار بود مطالب رو برام تایپ کنه این کار امکان پذیر نشد . امید وارم که بتونم جبران کنم.
الان ایام امتحانات است و امیدوارم که همه ی دانش آموزان در امتحانات موفق بشوند .
5 بهمن ماه مرحله ی اول المپیاد ریاضی است و برای تمام شرکت کنندگان در این امتحان آرزوی موفقیت دارم .

امیدوارم هرکجا که هستید موفق و پیروز باشید .

m_honarmand_j
11-01-2008, 00:02
سلام

16 رخ را در صفحه ی شطرنج می گذاریم . یک جفت از رخ ها را بد می نامیم هرگاه این دو رخ همدیگر را در صفحه تهدید کنند . حداقل تعداد جفت های بد برای این 16 رخ چند تا است ؟(مهره ی رخ در شطرنج می تواند کل سطر و ستونی را که در آن قرار دارد ، تهدید کند)

m_honarmand_j
11-01-2008, 00:08
سلام

هرمی با قاعده ی 9 ضلعی محدب داریم . تمام قطرهای قاعده و تمام یال های منتهی به رأس هرم را با دو رنگ مختلف رنگ می کنیم . (توجه کنید که اضلاع قاعده رنگ نمی شوند) ثابت کنید مستقل از رنگ آمیزی ما یک مثلث تک رنگ بوجود می آید.

pp8khat
12-01-2008, 22:25
آقای هنرمند،فکر کنم ملت از ترکیبات خسته شدن!
احتمال بلدید؟
منم یه چیزایی(ابتدایی) بلدم،بهتر نیست یه خورده از احتمال هم مسئله بذارید؟
ممنون

sanih
18-01-2008, 18:48
دوستان اینم احتمال:

اگر شش نقطه بطور تصادفي روي محيط يک دايره انتخاب کنيم.احتمال اينکه مثلثي که به وسيله سه نقطه اول ساخته مي شود با مثلثي که توسط سه نقطه دوم ساخته ميشود نطقه بخورد نداشته باشند چقدر است؟

mahdigh2
19-01-2008, 04:09
آیا میتوانید چندجمله ای زیر را تجزیه کنید؟


x^100+5x^99+2x+2

منبع:150 مساله پیشنهادی برای المپیاد

mahdigh2
19-01-2008, 04:11
اینعبارت حد اقل سه عامل داره.

ss_m_ss
19-01-2008, 21:29
X^99[x+5]+2[x+1] x^100+5x^99+2x+2

sanih
22-01-2008, 22:57
اگه کسی نمی خواد حلش کنه جوایشو بذارم.

m_honarmand_j
23-01-2008, 00:41
سلام

16 رخ را در صفحه ی شطرنج می گذاریم . یک جفت از رخ ها را بد می نامیم هرگاه این دو رخ همدیگر را در صفحه تهدید کنند . حداقل تعداد جفت های بد برای این 16 رخ چند تا است ؟(مهره ی رخ در شطرنج می تواند کل سطر و ستونی را که در آن قرار دارد ، تهدید کند)

جواب : تعداد رخ ها در سطر اول تا هشتم را به ترتیب a1,a2,…,a8 در نظر می گیریم و تعداد رخ ها در ستون اول تا هشتم را به ترتیب b1,b2,…,b8 در نظر می گیریم . تعداد رخ های بد در سطر اول برابر (P(a1,2 (انتخاب دو از a1) در سطر دوم برابر (P(a2,2 .... و در سطر هشتم برابر (P(a8,2 است . تعداد جفت های بد در ستون اول تا هشتم برابر (P(b1,2 و (P(b2,2 و .... و (P(b8,2 است .
حال داریم:
P(a1,2)+…+P(a8,2)+P(b1,2)+…+P(b8,2) = (a1*(a1-1)/2) + … + (b8*(b8-1)/2) = (1/2)*(a1^2+…a8^2+b1^2+…+b8^2 - (a1+…+a8+b1+…+b8))
حال با توجه به اینکه تعداد رخ ها در مجموع سطر ها مساوی تعداد رخ ها در مجموع ستون ها و برابر 16 است(a1+…+a8=b1+…+b8=16 ) از نامساوی حسابی_مربعی استفاده می کنیم .
با توجه به این نامساوی داریم:
x1^2 + … +xn^2 >= ((x1+…+xn)^2) / n
حال اگر به جای x1 تا a1، xn تا a8 و بار دیگر b1 تا b8 را قرار دهیم ، خواهیم داشت :
= (1/2) * ((a1^2+…+a8)^2 /8 + (b1+…+b8)^2 /8 -(a1+…+a8+b1+…+b8)) * (1/2) >= (a1^2+…+b8^2-(a1+…+b8))
16 = (1/2) * (32) = (1/2) * (32+32-32) = (1/2) * (((16^2)/8) + ((16^2)/8) - (16+16))
درنتیجه حداقل ما باید 16 جفت بد داشته باشیم که اگر این 16 رخ را در در قطر صفحه بچینیم می توانیم به خواسته ی مورد نظر برسیم.

m_honarmand_j
23-01-2008, 00:48
سلام

هرمی با قاعده ی 9 ضلعی محدب داریم . تمام قطرهای قاعده و تمام یال های منتهی به رأس هرم را با دو رنگ مختلف رنگ می کنیم . (توجه کنید که اضلاع قاعده رنگ نمی شوند) ثابت کنید مستقل از رنگ آمیزی ما یک مثلث تک رنگ بوجود می آید.

راهنمایی : رأس هرم دا در نظر بگیرید که به آن 9 یال منتهی می شود . طبق اصل لانه کبوتری حداقل 5 تا از این یال ها باید به یک رنگ باشند . مثلا آبی .این پنج یال و رأس های منتهی به آن ها را در نظر بگیرید . اگر دوتا از آنها با رنگ رنگ آبی به هم متصل باشند ، مسئله حل است . پس فرض می کنیم که این طور نباشد و ... . پس چیزی که می ماند این است که ثابت کنیم هر پنج رأسی را که انتخاب کنیم ، در بین آنها یک مثلث به وجود می آید و اثبات این هم آسان است و به شما واگذار می شود .

m_honarmand_j
23-01-2008, 00:57
سلام دوستان
از اینکه می بینم دو نفر از دوستان دست به کار شده و سوال گذاشته اند ، خوشحالم . اما در جواب دوست خوبم pp8khat در مورد خسته شدم دیگران از ترکیبیات و پیشنهاد ایشون در رابطه با احتمال می خواستم مطابی رو بگم .
اول اینکه دلیل شما برای اینکه دیگران از ترکیبیات خسته شدن چیه؟
اگر بخاطر عدم شرکت دیگران در بحث ها است ، این که چیز تازه ای نیست و از همان اول هم تعداد معدودی در بحث ها شرکت می کردند . اگر دلیل دیگری دارد لطفا بگید تا منهم بفهمم و اگه واقعا ملت از ترکیبیات خسته شدن ، بگید تا من هم خودم رو به زحمت نیندازم که البته من با نظر شما مخالف ام و فکر می کنم که ترکیبیات واقعا مبحثی شیرین و دوست داشتنی ای است .
در مورد احتمال هم باید بگم که مطرح نکردن سوالات احتمال از طرف من در اتاق ترکیبیات چند دلیل داشت :
1- در تاپیک ها ، تاپیکی با عنوان آمار وجود دارد و احتمال نیز مربوط به آمار است و من انتظار داشتم که سوالات احتمال در آنجا مطرح گردند . ولی متأسفانه این طور نشد و بخش مربوط به آمار چندان رونقی نداشته و از این به بعد به احتمال زیاد سوالات مربوط به احتمال را نیز در اتاق ترکیبیات خواهیم داشت .
2- دلیل بعدی دلیلی شخصی است که من شخصا اعتقادی به احتمال ندارم و از احتمال خوشم نمی آید . در منطق فازی که جایگزین منطق صفر و یک است و نزدیکی بیشتری با منطقی که ما انسانها در زندگی خود برای انتخاب هایمان به کار می بریم دارد و از آن برای هوشمندتر کردن ماشین ها استفاده شده است(این ها را گفتم که یک وقف فکرنکنید بی دلیل از احتمال بدم می آید و یا این که این منطق ساخته ی ذهن خودم است و هیچ استفاده ای هم ندارد در حالی که این منطق توانست کاری را که سالهای زیاد منطق صفر و یک برای هوشمند کردن ماشین ها نتوانست انجام دهد را ، انجام داد و کلا اگر کمی با آن آشنایی پیدا کنید فکر کنم شما هم با من هم عقیده شوید که این منطق از منطق صفر و یک قویتر و به منطق انسانها نزدیکتر است.)، احتمال به طور کلی رد می شود و بر اساس این منطق ذاتا در عالم هیچ احتمالی وجود ندارد. (اگر اطلاعات بیشتری در مورد این منطق خواستید می توانم به شما ارائه کنم زیرا تقریبا اطلاعاتم در این زمینه کامل است. )
3- دلیل بعدی این است که من خودم فکر می کنم در بخش احتمالات ظعیف هستم . به همین دلیل سعی کردم که در باب احتمال سوالی را مطرح نکنم و این کار را به دیگران که مجرب تر هستند واگذار کنم .

اما با وجود این دلایل وقتی که میبینم کسانی هستند که مشتاق احتمالات هستند و با توجه به اینکه در این زمینه می توان سوالات جالب و چالش انگیزی مطرح کرد ، نطرم رو عوض کردم و از این به بعد سوالاتی در این زمینه هم مطرح خواهم کرد . (البته با کمک دوستان به خصوص pp8khat )

m_honarmand_j
23-01-2008, 01:18
سلام دوست خوبم sanih
اگر سوالی را مطرح کردید و تا یک هفته ی بعد از مطرح کردن سوالی جوابی به سوال داده نشد ، اگر جواب سوال را خودتان می دانستید ، جواب را نیز لطف کرده و بگویید.
با تشکر.

sanih
23-01-2008, 14:27
امشب جواب سئوال رو میذارم .

pp8khat
24-01-2008, 10:17
سلام آقای هنرمند.
راستش من فکر کردم شما چون واسه المپیاد کامپیوتر می خوندید احتمال رو هم فول بودید چون احتمال و ترکیبات خیلی به هم نزدیکند...اما اعتماد نداشتن به موضوعی باعث میشه که آدم تو یادگرفتنش بی میل بشه..."امید ریاضی"که اصل کاربرد در زندگی و سر کار گذاشتن مردمه(یکی از مباحث احتمال)...تعیین کردن مقدار عادلانه پولی که باید بپردازید تا در یک مسابقه شرکت کنید مهم نیست؟به هر حال هرکی نظر خودش رو داره...
منم بعداً یه سری تعریف و مسئله از احتمال می ذارم...
Sanih جان!من گفتم یه چیزایی از احتمال بلدم...ولی سوال شما پیشرفته تر از این ها است!!!
من تا حالا فقط تونستم بفهمم که مثلث اول به 50 طریق ساخته میشه...بقیش رو بلد نیستم!

sanih
25-01-2008, 01:35
پیشرفته کجا بود این سئولات رو اگه بخوای روش حلش رو حفظ کنی اصلا نمی تونی حلش کنی باید دید ریاضی داشته باشی البته من به سواد شما جسارت نمی کنم و خودم رو شاگرد همه ی بروبچ پی سی میدونم.

sanih
25-01-2008, 01:36
دوستان این هم جواب سئوال احتمال:
[ برای مشاهده لینک ، لطفا با نام کاربری خود وارد شوید یا ثبت نام کنید ]

sanih
26-01-2008, 23:50
می خواد بشتر توضیح بدم؟

sanih
30-01-2008, 00:20
دوستان سئوال بذارید اگه نه دوباره از اون سئولات به ظاهر پیشرفته مذارم ها!

sanih
08-02-2008, 02:04
بابا یه سئوال بذارید دیگه خسته شدم

pp8khat
08-02-2008, 20:58
سلام...ببخشید خیلی.خیلی.خیلی....خیلی(n بار)سرم شلوغه...فرصت نکردم سوالا رو بذارم...

در ضمن آقای sanih :اولاً چرا می زنی؟ثانیاً:من واسه اون گفتم که سخته چون هنوز نه جبر و احتمال خوندیم و نه بخش آنالیز ترکیبی و احتمال ریاضی 2 رو(من سال دومم)...من هنوز فرق جایگشت و ترکیب رو نمی دونم...
راستی آقای هنرمند.فکر کنم بهتره من برم تو تاپیک آمار سوال ها رو بذارم تا شما به ترکیبات برسی..حله دوست عزیز؟ پس کسایی که سوال احتمال می خوان برن تو تاپیک آمار و احتمالات(البته فعلاً سرم شلوغه)

حالا این یه سوال رو داشته باشید:(منبع:مقدمه ای بر آمار و احتمالات:انتشارات دانشگاه تهران)
سوال: سه ماشین اتوماتیک قطعات مشابه اتومبیل را تولید می کنند.ماشین A به نسبت 40%،ماشین B به نسبت 25% و ماشین C به نسبت 35% از مجموع را تولید می کنند.به طور متوسط،10% از قطعات ساخته شده به وسیله ماشینA با مشخصات وفق نمی دهد و برای ماشین های B و C درصد های متناظر به ترتیب 5% و 1% است.
اگر یک قطعه،تصادفی از محصول روی هم ریخته شده انتخاب گردد و مشاهده شود که با مشخصات وفق نمیدهد،احتمال اینکه این قطعه به وسیله ماشین A تولید شده باشد،چند درصد است؟
(راهنمایی:این سوال با استفاده از قانون بیس قابل حل است)

sanih
08-02-2008, 23:06
چیزیت که نشد ؟ها من همیشه نمی زنم ولی خوب دیگه خیلی عصبانی شدم!

zahedy2006
09-02-2008, 02:41
سوال: سه ماشین اتوماتیک قطعات مشابه اتومبیل را تولید می کنند.ماشین A به نسبت 40%،ماشین B به نسبت 25% و ماشین C به نسبت 35% از مجموع را تولید می کنند.به طور متوسط،10% از قطعات ساخته شده به وسیله ماشینa با مشخصات وفق نمی دهد و برای ماشین های B و C درصد های متناظر به ترتیب 5% و 1% است.
اگر یک قطعه،تصادفی از محصول روی هم ریخته شده انتخاب گردد و مشاهده شود که با مشخصات وفق نمیدهد،احتمال اینکه این قطعه به وسیله ماشین A تولید شده باشد،چند درصد است؟
(راهنمایی:این سوال با استفاده از قانون بیس قابل حل است)
ذهنی: یک پنجاهم

pp8khat
09-02-2008, 14:38
ذهنی: یک پنجاهم

متاسفانه خیر...حتی نزدیک هم نیست.

zahedy2006
09-02-2008, 18:13
ذهنی
اینبار غیر ذهنی

راستی اون قضیه بیز است نه بیس (یه ساعت رفتم تو بحر base)
[ برای مشاهده لینک ، لطفا با نام کاربری خود وارد شوید یا ثبت نام کنید ]

غلط ، درستشم با خدا

pp8khat
09-02-2008, 20:19
اینبار غیر ذهنی

راستی اون قضیه بیز است نه بیس (یه ساعت رفتم تو بحر base)
[ برای مشاهده لینک ، لطفا با نام کاربری خود وارد شوید یا ثبت نام کنید ]

غلط ، درستشم با خدا

سلام.والا تو اون کتابه نوشته قضیه بیس(Bayes theorem)(یه کشیش بوده):11:
اما جواب!:5:

حل:اگر واقعه انتخابی یک قطعه ناقص را با D نمایش دهیم،باید احتمال P(A/D) برای قطعه انتخابی از ماشین A را تعیین کنیم،به فرض آنکه قطعه انتخابی ناقص باشد؛خواهیم داشت:
[ برای مشاهده لینک ، لطفا با نام کاربری خود وارد شوید یا ثبت نام کنید ]
حال P(D) یعنی احتمال به دست آوردن یک قطعه ناقص و می توان آن را به صورت مجموع احتمال های زیر نشان داد:
P(DA)=P(A)P(D/A)=(0.40)(0.10)=0.0400
+P(DB)=P(B)P(D/B)=(0.25)(0.05)=0.0125
+P(DC)=P(C)P(D/C)=(0.35)(0.01)=0.0035
=
0.0560
بنابراین:
P(A/D)=(0.40)(0.10)/0.0560
=0.714
که نشان میدهد به طور متوسط ماشین 71.4% از قطعات ناقص را تولید میکند.

(خودم هم نفهمیدم چی گفتم! راستی! شما تو رشته فیزیک چند واحد ریاضیات دارین؟)

zahedy2006
10-02-2008, 12:31
با سلام

حل:اگر واقعه انتخابی یک قطعه ناقص را با D نمایش دهیم،باید احتمال P(A/D) برای قطعه انتخابی از ماشین A را تعیین کنیم،به فرض آنکه قطعه انتخابی ناقص باشد؛خواهیم داشت:
حال P(D) یعنی احتمال به دست آوردن یک قطعه ناقص و می توان آن را به صورت مجموع احتمال های زیر نشان داد:
P(DA)=P(A)P(D/A)=(0.40)(0.10)=0.0400
+P(DB)=P(B)P(D/B)=(0.25)(0.05)=0.0125
+P(DC)=P(C)P(D/C)=(0.35)(0.01)=0.0035
=
0.0560
بنابراین:
P(A/D)=(0.40)(0.10)/0.0560
=0.714
که نشان میدهد به طور متوسط ماشین 71.4% از قطعات ناقص را تولید میکند.



فکر کنم جوابتون یک کم عجیب است.

توضیح: احتمالات شرطی - به احتمال قوی خودتون بلدبد ولی با این حال...

صورت سوال: احتمال اینکه قطعه معیوب ساخته ماشین A باشد
در صورت سوال احمتال به صورت شرط بیان شده است.
احتمال ساخته شدن به وسیله A به شرطی که معیوب باشد

اونجا که نوشته P A|D هم منظور همینه (پرانتز نگذاشتم)
P(D که همون احتمال خراب بودن است را که هر دو یکی در آورده ایم (البته من H گرفتم)

دقت کنید در جاگذاری در فرمول نیز یکسان عمل کرده ایم جز اینجا : P D|A
یعنی احتمال خراب بودن به شرط اینکه ماشین A آنرا تولید کرده باشد. می دانیم 40 درصد تولیدات مربوط به ماشین A است و از این 40 درصد ، 10 درصد خراب می باشند یعنی (100/40 * 100/10) ده درصدِ 40 درصد (اون علامت کسره است)

اما نتیجه: احتمال اینکه یک قطعه معیوب را ماشین A ساخته باشد مقدار جواب است. این چه نتیجه ای است که کتاب گرفته؟


راستی! شما تو رشته فیزیک چند واحد ریاضیات دارین؟
خیلی دقیق نمی دونم ولی بدون درس ریاضی و فیزیک 14 و با اون 20 تا

101170
11-02-2008, 16:36
من يه سوال دارم كه به صورت عكس درآوردم
اما چون زياد با كامپيوتر آشنايي ندارم نمي دونم عكس رو چطوري و كجا آپلود كنم لطفا توضيح مختصري بديد

zahedy2006
11-02-2008, 20:26
در سایت tinypic.com عکس را آپلود کنید. بعد آدرسش را در اینجا کپی کنید

101170
12-02-2008, 14:41
شكل زير از دو صليب با يك پايه با تركيب 13 مربع نمايش داده شده
خط CEشكل را به دو شكل با مساحت يكسان تبديل كرده طول BC چند است
[ برای مشاهده لینک ، لطفا با نام کاربری خود وارد شوید یا ثبت نام کنید ]

101170
12-02-2008, 21:23
يعني هيچكس نمي تونه جواب بده؟

101170
14-02-2008, 15:31
خودم جوابش رو بذارم؟

101170
15-02-2008, 14:48
[ برای مشاهده لینک ، لطفا با نام کاربری خود وارد شوید یا ثبت نام کنید ]

101170
16-02-2008, 20:57
:20:در دايره اي به قطر برابر 5 سانتي متر،يك چهارضلعي محاط كرده ايم و مي دانيم طول سه ضلع متوالي آن به ترتيب برابر 2، 3 و 4 سانتي متر است. طول ضلع چهارم اين چهارضلعي چقدر است؟

m_honarmand_j
21-02-2008, 12:10
سلام دوستان یک مدت نبودم .
راستش رو بخواهید کامپیوترم ویروسی شده و الان هم دارم از مدرسمون می نویسم.
اما تا آخر این هفته مشکل کامپیوترم حل می شه.
خب برسیم به تاپیک.
به نظر می رسد که تاپیک دارد از مسیر اصلی منحرف می شود . خدمت آقای 101170 باید بگویم که این تاپیک مخصوص ترکیبیات است و از گذاشتن سوالات هندسه در این تاپیک خود داری کنید.
خدمت آقای pp7khat بگوییم که مشکلی نیست و می تونید سوالات احتمال هم بگذارید. من احتمال تا حد خوبی بلد هستم و تنها از احتمال خوشم نمی آید.( چون برای المپیاد کامپیوتر می خواندم تا حد خوبی احتمالم را قوی کرده بودم.)
از دوست خوبم sanih هم می خواهم اگر امکان دارد پست های بی هدف (نمی دونم اصطلاح خوبی به کار بردم یا نه) نفرستند و اگر کسی مطلبی را متوجه نشود خودش خواهد گفت . و از اینکه می بینم ایشون اینقدر اشتیاق داردن که سوال حل کنند خوشحال می شوم و سی می کنم که سوال های خوبی قرار بدهم .
فعلان تا بعد.

mehdi_7070
24-02-2008, 23:10
سلام

تابع هموگرافیک دارای دو محور تقارن به شیب های 1 و 1- می باشد .

چرا شیب معادلات محور تقارن 1 و 1- می باشد؟

با تشکر

m_honarmand_j
04-03-2008, 10:14
سلام
بعد از چند روز بالاخره یک سوال جالب در مورد احتمال پیدا کردم.
دو تا تاس معمولی داریم . این دو تاس را پرتاب می کنیم و مجموع اعدد آمده را حساب می کنیم . چون اعداد 1 تا 6 روی تاس ها است پس حداقل مجموع 2 و حداکثر نیز 12 است و به احتمال 36/1 مجموع 2 می آید ، به احتمال 36/2 مجموع 3 می آید و ... و به احتمال 36/1 مجموع 12 می آید .
سوال: آیا امکان دارد دو تاس دیگر (می توانند دوتاس مشابه و یا متفاوت از هم باشند و باید با تاس معمولی فرق کنند و این تفاوق در اعداد روی تاس ها است) را طراحی کنیم که وقتی آنهارا با هم پرتاب می کنیم احتمال اینکه مجموع 2 بیاید 36/1 ، احتمال اینکه مجموع 3 بیاید 36/2 و ... و احتمال اینکه مجموع 12 بیاید 36/1 باشد؟ (دقت کنید که همه ی این احتمالات باید برقرار باشد)
سوال جالبی است . پیشنهاد می کنم روش فکر کنید . اگر هم توضیح بیشتری در مورد سوال خواستید بگویید.

zahedy2006
04-03-2008, 16:24
راجع به این سوال آخر:

سریعترین جوابی که می تونستم بدهم (زیر 10 ثنیه فکر کردم) و البته احمقانه ترین جواب

چون گفتید تاس ها مشابه و معمولی نیستند
یکی را از 0 تا 5 و دیگری را از شما 2 تا 7 شماره گذاری می کنیم حالا شد همان تاس قبلی (از لحاظ مجموع اعداد و حالات انتخاب)

ببخشید که مسخره بود !!

mofidy1
07-03-2008, 13:32
با سلام

آقا مهدی عزیز، تابع هموگرافیک (ax+b)/(cx+d) دارای دو محور تقارن به معادلات y=-x+(a-d)/c و y=x+(a+d)/c است که این دو شیب های 1- و 1 دارند.

موفق باشید.

17 اسفند 1386

mehdi_7070
07-03-2008, 17:55
با سلام

آقا مهدی عزیز، تابع هموگرافیک (ax+b)/(cx+d) دارای دو محور تقارن به معادلات y=-x+(a-d)/c و y=x+(a+d)/c است که این دو شیب های 1- و 1 دارند.

موفق باشید.

17 اسفند 1386

خیلی ممنون جناب آقای مفیدی :11:

اگه میشه دلیل اینکه چرا این دو معادله ، معادلات محور تقارن هستند را هم بنویسید :10:

m_honarmand_j
10-03-2008, 00:52
سلام
دوست خوبم zahedy2006
اول از همه بگم که اعداد روی تاس ها باید طبیعی باشند و من این نکته رو فراموش کرده بودم بگم .
حتی با فرض اینکه صفر رو هم داشته باشیم باز هم جواب شما درست نیست .
البته نباید شما دلسرد بشید و هر جوابی که به ذهنتون اود رو بگید . چون اینجا هدف ما ضایع کردن دیگران و یا نمره دادن و امثال این ها نیست و فقط تلاش ما برای ارتقای سطح سواد دیگران و خودمان است .
برای غلط بودن جواب شما فقط یک نمونه رو می گم .
در پرتاب دو تاس معمولی احتمال اینکه مجموع 3 بشدو برابر 36/2 است زیرا تنها دو حالت (1و2) و (2و1) به مجموع 3 منتهی می شوند در حالی که در پرتاب دو تاس طراحی شده ی شما احتمال اینکه مجموع 3 بشود 36/3 است زیرا حالت های (3و0) و (2و1) و (1و2) به مجموع 3 منتهی می شوند .
Ok؟

m_honarmand_j
10-03-2008, 00:56
سلام
سوال:
مجموع انتخاب 0 از 3n + انتخاب 3 از 3n + ... انتحاب 3k از 3n + ... + انتخاب 3n از 3n چقدر می شود ؟

jinn's king
12-03-2008, 20:32
با سلام
يكي بهم بگه ميشه دو معادله سه مجهول رو حل كرد اگه ميشه لطفا اين و حل كنين
با انجام عمليات

./1x + y + 3z=100
x + y + z=100

پيشاپيش متشكرم

اولي يك دهم ايكس مي باشد و اخري صد است يعني
100=سه زد+ايگرگ+ يك دهم ايكس

yugioh
12-03-2008, 23:01
دو معادله 3مجهول رو نمیشه حل ب جواب یکتا کرد . مثلا جواب شما برخورد دو صفحه است که یک خطه( ممکنه یک صفحه(معادله یکسان) یا بدون جواب (دوصفحه موازی) بشه. ولی ممکنه سوال در مجموعه اعداد طبیعی جواب یکتا داشته باشه یا در بعضی موارد بشه یک مجهول از سه تا رو دراورد( خط موازی محور)

man7tomani
12-03-2008, 23:04
نمیدونم ولی فر کنم اینجوری باشه!
[ برای مشاهده لینک ، لطفا با نام کاربری خود وارد شوید یا ثبت نام کنید ]
بقیش رو که دیگه بلدی ها؟

man7tomani
12-03-2008, 23:07
دو معادله 3مجهول رو نمیشه حل ب جواب یکتا کرد . مثلا جواب شما برخورد دو صفحه است که یک خطه( ممکنه یک صفحه(معادله یکسان) یا بدون جواب (دوصفحه موازی) بشه. ولی ممکنه سوال در مجموعه اعداد طبیعی جواب یکتا داشته باشه یا در بعضی موارد بشه یک مجهول از سه تا رو دراورد( خط موازی محور)
عزیز میشه بیشتر توضیح بدی؟

yugioh
12-03-2008, 23:18
عزیز میشه بیشتر توضیح بدی؟

خواهش می کنم.
هر معادله3 متغیره خطی مثل این یه صفحه رو توصیف می کنه بسته به وضعیت 2 صفحه با هم منطبق یا موازی یا مطقاطع نقاط مشترک دو صفحه جوابند در نتیجه با دو صفحه به جواب نمی رسیم. چیزی که شما گفتین درسته ولی فقط معادله یکی از صفحاتیه که جواب عضوشونه که این صفحه بی نهایت عضو داره.

jinn's king
12-03-2008, 23:32
بالاخره مقادير x y , z را به طور واضح مشخص كن من صحفه زياد سر در نمي ارم

man7tomani
12-03-2008, 23:34
خواهش می کنم.
هر معادله3 متغیره خطی مثل این یه صفحه رو توصیف می کنه بسته به وضعیت 2 صفحه با هم منطبق یا موازی یا مطقاطع نقاط مشترک دو صفحه جوابند در نتیجه با دو صفحه به جواب نمی رسیم. چیزی که شما گفتین درسته ولی فقط معادله یکی از صفحاتیه که جواب عضوشونه که این صفحه بی نهایت عضو داره.
بدلیل فاقد بودن انجمن از دکمه ی تشکر من مجبورم بطور تایپی از شما تشکر کنم
اگه ممکنه یه مثال برام بزنین
اگه میشه داخل Word لطفا
اینجور شاید این آقا هم بیشتر متوجه بشن و هم من
راستش نفهمیدم
یهنی چی بی نهایت عضو داره؟!
اگه 3 صفحه باشن جواب میده؟

yugioh
12-03-2008, 23:54
نیازی به تشکر نیست. ولی در مورد سوال jinn's king : گفتم نمیشه جواب یکتا بدست آورد جواب یه مجموعه است مثلا:
(x=0 z=0 y=100, x=(100/1.45),y=0, z=(45/1.45, .............. همه جوابند.
در مورد اینکه فحه و اینا منظور چیه این طور بگم: شما یه نقطه رو در سه بعد در دستگاه مختصات با سه مولفه ی x,y,z معلوم می کنید. مثلا(1,1,1) یه خط یه مجموعه نقطه است یه صفحه هم همین طور.دو تا معادله ای که نوشته بودن معادله دو تا صفحه بود . و نقاطی که x,y,x اونها تو معادله صدق کنه نقاط عضو اون صفحه اند. وقتی یه نقطه جواتب دستگاه معادله باشه یعنی جواب تک تکمعادلات هم هست یعنی رو یه هر دو صفحه هست پس باید روی نقاط مشترک دو صفحه باشه. خب دوتا صفحه در فضا چند نوع موقعیت نصبت به هم دارن؟ 3نوع:
1.منطبق: یعنی عملا یه صفحه اند. پس نقاط مشترکشون(جواب دستگاه معادلات) میشه یه صفحه مثل:x+y+z=1 ,2x+2y+2z=2
2. موازی: یعنی هیچ نقطه مشترکی ندارند (دستگاه بدون جواب). مثل:x+y+z=1 ,x+y+z=2
3. متقاطع:یعنی در یک خط مشترکند (یک خط میبشه جواب): مثل x+y+z=1,2x+y+z=1
بازهم در خدمتم.

nadernader172
14-03-2008, 02:00
این دستگاه بینهایت جواب دارد که جوابهای آن همگی روی یک خط راست در دستگاه مختصات فضایی قرار دارد و لازمۀ بدت آوردن معادلۀ این خط اینست که مسائل مربوط به معادلات خط و صفحه را بدانید.
به صورت جبری میتوان اینگونه گفت:
شما به صورت دلخواه یکی از متغیرها را معلوم فرض کنید مثلاً: x=a
بنابراین:
z=(9/20)a
y=100 - (29/20)a
به ازای هر عددی که بجای a قرار میگیرد یک جواب برای x یکی برای y و یکی برای z جواب خواهیم داشت.

sherlockholmz
16-03-2008, 09:08
بالاخره مقادير x y , z را به طور واضح مشخص كن من صحفه زياد سر در نمي ارم
سلام،

همانطور كه دوستان توضيح دادند، n معادله ،m مجهول با شرط m>n بي نهايت جواب دارد.اما يك سري از چنين معادلاتي به نام معادلات سيال شناخته مي شوند كه در مواردي قابل حل هستند.معادلات سيال دو شرط دارند:
1-m=n+1
2-مجهولات همگي اعداد صحيح هستند.
با اين دو شرط گاهي مي توان جواب منحصر به فردي براي معادله بدست آورد.
موفق باشيد.

sherlockholmz
16-03-2008, 09:50
معادله ايي كه شما نوشته ايد يكي از معروفترين مسائل از اين دست است.با ضرب معادله اول در 10 ،دستگاه زير بدست مي آيد كه آنرا برايتان حل مي كنم:

[ برای مشاهده لینک ، لطفا با نام کاربری خود وارد شوید یا ثبت نام کنید ]


سوالي بود در خدمتم
موفق باشيد.

_Mohsen_
22-03-2008, 10:15
دليلشو من بت ميگم ... ساده ترين تابه هموگرافيكو در نظر بگير ! Y= 1 / x (گرچه هميشه معادله تابع همو گرافيك تو دستگاهي كه از انتقال محور هاي مختتصات به مركر تقارن تابع هموگرافيك بدست مي ياد همون Y= 1 / x) اميد وارم اقلاً در مورد مركز تقارنش سوان نكني اگه از دوران محور هاي مقتصاد چيزي مي دوني سعي كن معادله ساده ترين تابع همو گرافيكو تو دستگاهي كه از دوران محورهاي مقتصادت به انازه 45 درجه در جهت مثلاتي برست ميادو بدست بياري و تحقيق كني كه خط x=0 تو دسنگاه جديد محور تقارن مهادله جديده كه نا خودآگاه وقتي از درون دستگاه قديمي به فرازو نشيب هايي كه طي كردي نگاه مي كني كي بيني كه آي دل غافل ، تابع همو گرلفيك دو تا محور تقارن داره!

_Mohsen_
22-03-2008, 23:53
تا اونجا كه ما ميدونم دستور ساروس فقط دترمينان ماتريس مربعي 3*. رو بدست ميده نه n*n

m_honarmand_j
25-03-2008, 02:36
سلام دوستان
عید بر همه ی شما مبارک باد.
می بینم که کسی راه حلی برای مسئله ی تاس ها نداده .
من اننتظار نداشتم کسی این سوال و حل کنه و راه حلی هم که من دارم کمی دور از ذهن است . ولی گفتم شاید کسی پیدا بشه و راه آسان تری را پیشنهاد بده . من با توابع مولد این مسئله را حل کردم . شما ها هم با این راه روی مسئله فکر کنید ببینید می تونید حلش کنید .

m_honarmand_j
25-03-2008, 02:43
سوال:
حداکثر تعداد مثلث که ممکن است در یک تورنومنت n راسی بوجود بیاید چند تا است؟
(یک تورنومنت عبارت است از n راس که بین هر دو راس دقیقا یک یال وجود دارد وهر راس نمایده ی یک فرد یا گروه می تواند باشد که در این تورنومنت شرکت کرده اند و یال ها جهت دار اند به طوری که اگز فرد a از b برده باشد آنگاه جهت یال ab به سمت b حواهد بود و یک مثلث یعنی اینکه فرد a از فرد b برده، فرد b از فرد c و فرد c از فرد a برده است . )

sanih
04-04-2008, 20:43
به نام خدا



سلام بعد از نمی دونم چند ماه دوباره هوس نوشتن به سرم زد:



یه سئوال جالب دارم کسی میتونه حل کنه؟ حتما میتونید اگه هم نه....نه بابا میتونید.



سئوال از این قراره که هشت جعبه داریم که تو هرکدومشون شش تا توپ هست.میخواهیم تمام این توپ هارو طوری رنگ بزنیم که هر دو توپ در یک جعبه رنگ های متفاوتی داشته باشند و همچنین هر جفت رنگ حداکثر در یک جعبه ظاهر شده باشند.حد اقل رنگهایی که میتونیم استفاده کنیم چقدر است؟




بعدا" خودم جواب رو میذارم

sanih
04-04-2008, 20:57
همون طور که دوستمون اشاره کرد با گذاشتن شرط هایی میشه این معدله ها رو حل کرد مثلا با شرط صحیح و نا منفی بودن واسه معادله زیر می شه چندتا جواب محدود پیدا کرد:
پدری 10 ملیون پول دارد و میخواهد به 3 پسرش پول قرض دهد پدر تواول های 1ملیونی دارد. میبینید که شبیه مسائل ریاضیات گسسته است

یاحق

Sub007
05-04-2008, 14:57
معادله ايي كه شما نوشته ايد يكي از معروفترين مسائل از اين دست است.با ضرب معادله اول در 10 ،دستگاه زير بدست مي آيد كه آنرا برايتان حل مي كنم:

[ برای مشاهده لینک ، لطفا با نام کاربری خود وارد شوید یا ثبت نام کنید ]


سوالي بود در خدمتم
موفق باشيد.

برای حل معادلات سیاله درجه اول در مجموعه اعداد صحیح، می‌تونید از هم‌نهشتی هم استفاده کنید که سریعتر و قابل فهم‌تر هست. البته در این مورد به علت رُند بودن اعداد تفاوت چندانی در سرعت محاسبات وجود ندارد.

khatarat
07-04-2008, 15:27
کی می تونه این دستگاه سه معادله سه مجهول رو حل کنه
a^2=x^2+y^2+z^2+x1^2-2xx1
b^2=x^2+y^2+z^2+y1^2-2yy1
c^2=x^2+y^2+z^2+z1^2-2zz1
تو این دستگاه x y z مجهولند و بقیه عدد ثابت هستند.

sanih
08-04-2008, 00:01
error.........

sanih
08-04-2008, 00:10
error.........

Iron
08-04-2008, 09:04
خط اول را از خط دوم کم می کنیم یک رابطه خطی بدست میاد که از اون y برحسب x بدست میاد. بعد خط اول رو از خط سوم کم می کنیم تا رابطه خطی بدست بیاد که z برحسب x بدست بیاد. در انتها y و z رو برحسب x توی رابطه اول قرار می دیم تا یه معادله درجه دوم بدست بیاد و اونو حل می کنیم.

sanih
08-04-2008, 23:17
دوستان جواب سئوال رو بذارم؟

mahsa9134
09-04-2008, 19:53
ثابت كنيد كه در هر دسته 10 نفري يا زيرمجموعه اي مركب از 3 بيگانه وجود دارد يا زيرمجموعه اي مركب از 4 نفر كه يكديگر را مي شناسند/

sanih
13-04-2008, 21:05
دوست من،من این سئوال رو از چند نفر پرسیدم ولی اونا منظور مسئله رو هم نفهمیدن.

اگه میشه منبعت رو بگو و بگو از کجا آوردی تا شاید بتونم کمک چون این مسئله ذهن خودمم رو هم خیلی مسغول کرده

mahsa9134
14-04-2008, 18:11
دوست من،من این سئوال رو از چند نفر پرسیدم ولی اونا منظور مسئله رو هم نفهمیدن.

اگه میشه منبعت رو بگو و بگو از کجا آوردی تا شاید بتونم کمک چون این مسئله ذهن خودمم رو هم خیلی مسغول کرده
سلام
از توجهت ممنونم
منبعش اينه:
رياضيات گسسته مقدماتي
تاليف و.ك.بالاكريشنان
ترجمه دكتربيژن شمس
دكترمحمدعلي رضواني
انتشارات فاطمي
چاپ هفتم 1385

تركيبيات
تمرين 1-75 صفحه 83

mahsa9134
15-04-2008, 10:52
ميشه به اين سوال منم جواب بديد مربوط به لانه كبوتره؟
ثابت كنيد كه در هر دسته 10 نفري يا زيرمجموعه اي مركب از 3 بيگانه وجود دارد يا زيرمجموعه اي مركب از 4 نفر كه يكديگر را مي شناسند
من اينو واسه 4 شنبه لازم دارم

s_keivani
29-04-2008, 23:01
صفحه مختصات x,y را در نظر بگیرید.
یک جسم روی نقطه 0,0 قرار دارد.می خواهیم این جسم را به نقطه (m,n) منتقل کنیم.
بدین منظور حرکت روی صفحه از یکی از معادلات زیر تبعیت می کند:

x,y>>>>x+1,y
or
x,y>>>>x,y+1
or
x,y>>>>x+1,y+1

یعنی هر حرکت یا به سمت راست است یا بالا و یا به صورت قطری به شمال شرق !
به چند طریق می توان این جسم را به نقطه (m,n) برد ؟

pp8khat
15-05-2008, 16:11
یک مسئله نسبتاً ساده ولی جالب از احتمال:
کلاسی 7 نفر دانش آموز دارد.معلم این کلاس از دانش آموزان امتحانی می گیرد.روش معلم این کلاس برای تصحیح اوراق به این صورت است که پس از تمام شدن امتحان ورقه ها را از دانش آموزان گرفته و آنها را مخلوط می کند(بر می زند) سپس به هر دانش آموز به طور اتفاقی یک ورقه می دهد.

الف)احتمال اینکه حداقل یک دانش آموز ورقه امتحانی خودش را دریافت کند،چند درصد است؟

ب) اگر سه نفری که موقع امتحان غیبت کرده بودند(!!) نیز به جمع امتحان دهندگان اضافه شوند و امتحان بدهند،(یعنی 10 نفر امتحان بدهند) آیا احتمال به دست آمده در قسمت "الف" کاهش پیدا می کرد؟

ج)اگر از آن 7 نفر 3 نفر دیگر نیز موقع امتحان غیبت می کردند،(یعنی 4 نفر امتحان می دادند)آیا احتمال به دست آمده در قسمت "الف" افزایش می یافت؟

د) نسبت احتمال قسمت "الف" هنگامی که 4 نفر امتحان دهند به احتمال قسمت "الف" هنگامی که 10 نفر امتحان دهند را به دست آورید.

موفق باشید.

m_honarmand_j
15-05-2008, 23:20
سلام
یک مدت طولانی نبودم و دلایلش هم شخصی ه .
بگذریم.
امسال یکی از سخترین دوره های المپیاد ریاضی در مرحله ی دوم بود 0 ( البته به نظر من و دوستانم )
روز اول تقریبا سوالات خوب بودن و سطح سوالات خیلی بالا نبود . البته سوال 3 بد نبود و تقریبا سخت ترین سوال روز اول بود . ولی روز دوم سوال اول نظریه ی اعداد نسبتا سخت و اکثر راه حل هایی که من برای این مسئله دیدم سطح بالا بودن و در سطح مرحله ی دوم واقعا نبودن . سوال 4(سوال دوم روز دوم) که ترکیبیات بود واقعا سخت بود و نمی دونم آیا اصلا توی، کل کشور تعداد دورقمی از افراد تونستند این مسئله رو حل کنند یا نه . سوال اخرم که من خودم نرسیدم روش فکر کنم ولی از کسی که حل کرده شنیدم که می گفت خیلی سخت بود و احتمالا این مسئله رو هم تعداد زیادی حل نکرده باشن .(اون کسی که حل کرده بود از طلا های سال پیش ه که تو دوره ی انتخابی تیم ه)
فعلا خداحافظ

sherlockholmz
17-05-2008, 15:58
یک مسئله نسبتاً ساده ولی جالب از احتمال:
کلاسی 7 نفر دانش آموز دارد.معلم این کلاس از دانش آموزان امتحانی می گیرد.روش معلم این کلاس برای تصحیح اوراق به این صورت است که پس از تمام شدن امتحان ورقه ها را از دانش آموزان گرفته و آنها را مخلوط می کند(بر می زند) سپس به هر دانش آموز به طور اتفاقی یک ورقه می دهد.

الف)احتمال اینکه حداقل یک دانش آموز ورقه امتحانی خودش را دریافت کند،چند درصد است؟

ب) اگر سه نفری که موقع امتحان غیبت کرده بودند(!!) نیز به جمع امتحان دهندگان اضافه شوند و امتحان بدهند،(یعنی 10 نفر امتحان بدهند) آیا احتمال به دست آمده در قسمت "الف" کاهش پیدا می کرد؟

ج)اگر از آن 7 نفر 3 نفر دیگر نیز موقع امتحان غیبت می کردند،(یعنی 4 نفر امتحان می دادند)آیا احتمال به دست آمده در قسمت "الف" افزایش می یافت؟

د) نسبت احتمال قسمت "الف" هنگامی که 4 نفر امتحان دهند به احتمال قسمت "الف" هنگامی که 10 نفر امتحان دهند را به دست آورید.

موفق باشید.


مسئله را بصورت معكوس حل مي كنيم:
الف)احتمال اينكه هيچكس برگه خود را بدست نياوردعبارتست از:

[ برای مشاهده لینک ، لطفا با نام کاربری خود وارد شوید یا ثبت نام کنید ]


ودر نتيجه احتمال آنكه حداقل يكي برگه خود را بدست آورد:

[ برای مشاهده لینک ، لطفا با نام کاربری خود وارد شوید یا ثبت نام کنید ]


باتوجه به فرمول كلي زير،هر چه قدر تعداد نفرات امتحان دهنده بيشتر باشد، احتمال آنكه حداقل يكي برگه خود را بدست آورد بيشتر ميشود:

[ برای مشاهده لینک ، لطفا با نام کاربری خود وارد شوید یا ثبت نام کنید ]


مثلا" براي 10 نفر:

[ برای مشاهده لینک ، لطفا با نام کاربری خود وارد شوید یا ثبت نام کنید ]

ج)نه خير كاهش مي يافت!


[ برای مشاهده لینک ، لطفا با نام کاربری خود وارد شوید یا ثبت نام کنید ]

د)خواهيم داشت:


[ برای مشاهده لینک ، لطفا با نام کاربری خود وارد شوید یا ثبت نام کنید ]

شما هم موفق باشيد.

دوست ریاضی
04-08-2008, 17:09
یک شبکه mxn و سه رنگ داریم می خواهیم هر ضلع از شبکه را با یکی از لین سه رنگ چنان رنگ کنیم که هر مربع دو مربع کوچک دو ضلع از یک رنگ دو ضلع از رنگ دیگر داشته باشد. این کار به چند طریق امکان پذیر است؟

دوست ریاضی
04-08-2008, 17:20
سلام
یک مدت طولانی نبودم و دلایلش هم شخصی ه .
بگذریم.
امسال یکی از سخترین دوره های المپیاد ریاضی در مرحله ی دوم بود 0 ( البته به نظر من و دوستانم )
روز اول تقریبا سوالات خوب بودن و سطح سوالات خیلی بالا نبود . البته سوال 3 بد نبود و تقریبا سخت ترین سوال روز اول بود . ولی روز دوم سوال اول نظریه ی اعداد نسبتا سخت و اکثر راه حل هایی که من برای این مسئله دیدم سطح بالا بودن و در سطح مرحله ی دوم واقعا نبودن . سوال 4(سوال دوم روز دوم) که ترکیبیات بود واقعا سخت بود و نمی دونم آیا اصلا توی، کل کشور تعداد دورقمی از افراد تونستند این مسئله رو حل کنند یا نه . سوال اخرم که من خودم نرسیدم روش فکر کنم ولی از کسی که حل کرده شنیدم که می گفت خیلی سخت بود و احتمالا این مسئله رو هم تعداد زیادی حل نکرده باشن .(اون کسی که حل کرده بود از طلا های سال پیش ه که تو دوره ی انتخابی تیم ه)
فعلا خداحافظ
سلا امیدوارم تو امتحانت موفق بوده باشی:46: لطفا اون سوال ترکیبیات و نظریه اعداد رو اگر یادت هست جز سوالات مطرح کن . این سوالات مربوط به کدوم المپیاد ؟

Armin 2008
17-08-2008, 17:17
5 تا آدم دور یک میز گرد به چند حالت میتونن بشینن

Maxwell_1989
17-08-2008, 18:16
جواب:n شی متمایز را به (n-1) فاکتوریل طریق می توان دور یک دایره (یا بیضی یا ... ،فرقی ندارد) قرار داد.مثلا a,b,c را به دو طریق می توان دور دایره قرار داد:abc,acb
پس جواب سوال شما میشه:!4=24
با تشکر

Amin4250
26-08-2008, 15:01
ميشه به اين سوال منم جواب بديد مربوط به لانه كبوتره؟
ثابت كنيد كه در هر دسته 10 نفري يا زيرمجموعه اي مركب از 3 بيگانه وجود دارد يا زيرمجموعه اي مركب از 4 نفر كه يكديگر را مي شناسند
من اينو واسه 4 شنبه لازم دارم

افراد نقطه و رابطه دوستي يال است.
يك نفر را در نظر مي گيريم.حد اكثر نا آشنايان او 3 نفر است زيرا اگر 4 نفر باشد بايد همه آنها يكديگررا بشناسند تا 3نفر نا آشنا با هم ايجاد نشود كه در اين صورت مسئله حل است.حال اگر 3 نفر نا آشنا داشته باشد 6 نفر آشنا دارد و ما مي دانيم كه از هر 6 نفر 3نفر هستند كه يا دو به دو با هم آشنايند يا 3نفر كه دو به دو نا آشنايند.

m_honarmand_j
02-09-2008, 11:02
سلام بر همه ی دوستان
شرمنده که یه مدت نبودم و احتمالان تا مدت مدیدی هم نخواهم بود . چون من الان پیشم و باید برای کنکور بخونم و واقعا وقت نمی کنم که به این مسائل برسم . خیلی خوشحالام در زمانی که من نبودم این اتاق نخوابید و کسانی بودن که مطلب بدن . از همتون ممنونم.
فعلا خداحافظ.

m_honarmand_j
02-09-2008, 11:08
یکی از دوستان سوال مرحله ی دوم ریاضی رو که گفته بودم سخت بود و خواسته بود. اینم سوال:
در یک کشور می خوان شماره تلفن های 9 رقمی درست کنن و مجازند که از اعداد 1 تا 9 استفاده کنن . با این شرایط که هر دو شماره یا باید در یک رقم بیش از دو واحد اختلاف داشته باشند (مثلا 111111111 و 111111113)و یا در بیش از یک رقم اختلاف داشته باشند (مثلا 111111111 و 111111122). حئاکثر چند تا شماره ی 9 رقمی با این شرایط می توان ساخت و این تعداد شماره رو به چند طریق می توان ساخت؟

m_honarmand_j
02-09-2008, 11:16
صفحه مختصات x,y را در نظر بگیرید.
یک جسم روی نقطه 0,0 قرار دارد.می خواهیم این جسم را به نقطه (m,n) منتقل کنیم.
بدین منظور حرکت روی صفحه از یکی از معادلات زیر تبعیت می کند:

x,y>>>>x+1,y
or
x,y>>>>x,y+1
or
x,y>>>>x+1,y+1

یعنی هر حرکت یا به سمت راست است یا بالا و یا به صورت قطری به شمال شرق !
به چند طریق می توان این جسم را به نقطه (m,n) برد ؟


سلام

من وقت ندارم رو این سوال فکر کنم اما به عنوان راهنمایی فکر کنم با نوشتن روابط بازگشتی بشه این مسئله رو حل کرد. وقتی می خوایم بریم خونه ی (m,n) باید از یکی از خونه های (m-1,n) , (m,n-1), (m-1,n-1) که اگه روابط بازگشتیشونو بنویسیم و چند تا مقدار اولیه رو بدست بیاریم فکر کنم راحت بشه جوابو پیدا کرد. به احتمال زیاد باید راه حل های دیگه ای هم باشه ولی فعلا این راه به ذهنم رسید.

m_honarmand_j
02-09-2008, 11:47
راستی یک نفر از من در مورد المپیاد کامپیوتر سوال کرده بود ، من id ایشونو یادم نیومد تا بخوام بهشون جواب بدم به همین دلیل اینجا بهشون جواب می دم شاید به درد کسایه دیگه هم خورد :
در ادامه ی چیزایی که بهتون گفتم شما باید قسمت استقرای کتاب الفبای المپیاد های ریاضی و کامپیوتر رو حتما بخونید و سوالاشو حل کنید. سوالات ترکیبیات کتاب المپیاد های لینینگراد و هم حتما حل کنید . از کتاب المپیاد های شوروی هم قسمتهای ترکیبیاتشونو حل کنید (البته شوروی بیشتر به المپیاد ریاضی می خوره اما چیزای خوب هم داره که به درد کامپیوتر بخوره)
حتما برای مرحله ی اول حداقال سوالات مرحله های اول 10 سال اخیرو حل کنید. برای مرحله دوم هم همچنین.کتاب ترمیبیات خوشخوان هم خوبه البته اگه کتاب استراتژی و کتاب الفبا و کتاب فاطمی و اونایه دیگه که گفتم و بخونید میبینید که اکثر سوالات خوشخوان براتون تکراری ه . برای الگوریتم هم اگه کتاب معماههای الگوریتمی و بخونید یکم ذهنتون الگوریتمی می شه اما اگه می خواهید الگوریتم کار کنید کتابی هست معروف به C-L-R-S که اسمشو دقیق یادم نمیاد . احتمالا اسمش آشنایی با الگوریتم باشه . البته یه کتاب هم هست که اسمش اگه درست یادم باشه آشنایی با الگوریتم به زبان ++C ه که کتاب خوبی ه فط یکم قدیمی ه و نویسندش جان نیپولیتیان ه و ترجمه شده و اگه تونیتید یه کتاب هست که اگه مدرستون معلمی براتون اورد از اون باید بگیرید که این کتاب الگوریتم معروف به کریتیو ه و کتابش بیرون نیست و باید از یکی بگیرید و کپی کنید . برای گراف هم کتاب آشنایی با نظریه ی گراف نوشته ی دوگلاس ب وست ه که اگه اینگلیسیشو گیر بیارید بهتره .
دیگه من فعلا چیزی یادم نمی آد . اگه مشکلی داشتید بپرسید.

Maxwell_1989
11-09-2008, 15:46
سوالتون آنالیزیه.نباید(دوستانه!)تو اتاق ترکیبیات مطرحش کنید.

chessmathter
11-09-2008, 23:27
تعداد حالت هایی که میشه 8 تا وزیر تو خونه های شطرنج گذاشت که هیچ کدوم همدیگرو تهدید نکنن چند تاست؟
من خودم بعد از یه ایده سوال و به این convert !کردم
تعداد توابع از {1,2,3,4,5,6,7,8} به {1,2,3,4,5,6,7,8} که یک به یک و پوشاست و جمع مولفه اول و دوم و تفاضل مولفه دوم از مولفه اول هر زوج مرتبش متفاوت باشه؟
مثلا این یکیشه
{(1,4),(2,6),(3,1),(4,5),(5,2),(6,8),(7,3),(8,7)}
{15,10,14,7,9,4,8,5} جمع مولفه های هر زوج مرتب
{2,4,3-,3,1-,4,2-,1-} تفاضل مولفه دوم از مولفه اول
فعلا اینجا هنگ کردم :31::13:

اگه کسی تونست بقیه ایده منو بره و حل کنه بگه اگه هم راه حل سوال و میدونه یا حل کرده بگه(سوال اصلی همون صفحه شطرنج ست)

Maxwell_1989
12-09-2008, 14:35
خواهشا لطف کنید سوالات مربوط به آنالیز رو در اتاق حساب دیفرانسیل و انتگرال مطرح کنید و جواب بدید.نه توی اتاق ترکیبیات.

shape
12-09-2008, 14:54
اتفاقا به نظر من این یه سوال ترکیبیات هست که یه ایده تونسته شکلش رو عوض کنه
حالا این چه ربطی به حساب دیف داره؟

chessmathter
12-09-2008, 15:01
منم واسه همین اینجا گذاشتم ولی برای کانورتش تو حساب دیف هم گذاشتم
حالا خواهشن اگه خواستید همفکری کنید هنگم توش!!!

tarane
29-09-2008, 11:03
سلام دوستان دو تا سوال داشتم :19:

1- چرا منفی در منفی مثبت میشود؟

2- جذر چگونه بوجود آمد ؟
:41:
اگر کسی میدونه دریغ نکنه ممنون میشم :11:

Maxwell_1989
29-09-2008, 16:05
ممنون از سوالای جالبتون خواهر(؟) ترانه!
در مورد سوال اولتون عرض کنم که:
چون دو طرف یه تساوی باید هم علامت باشن.اینطوری میشه:
ببینید معادله "منفی بی در منفی آ=منفی سی"رو با در نظر گرفتن a,b,c با علامت مثبت،در نظر بگیرید.
می تونیم تبدیشل کنیم به"منفی یک در بی در منفی یک در سی=منفی یک در آ"حالا شما از هر طرف این تساوی یک منفی حذف کنید.اینجاست که به تناقض می خوریم.ویک عدد مثبت یعنی سی با یک عدد منفی یعنی منفی آبی برابر میشه.(البته این روش به شرطی درسته که قبول کنیم مثبت در مثبت میشه مثبت.امیدوارم شما قبول داشته باشید!)
برای سوال دوم باید بگم:
احتیاج بشر باعث کشف جذر شد!مثلا معادله x+1=0 منجر به معرفی اعداد منفی شد.x^2-4=0 (مثلا) هم موجب شد تا یه چیزی به اسم ریشه دوم اختراع بشه(برای یافتنx) یا مثلا وقتی دلتای معادله ی درجه دو منفی شد اعداد موهومی و از اونجا مختلط بوجود اومدن.
بازم سوالی بود در خدمتم.
با تشکر.

chessmathter
29-09-2008, 22:10
سلام دوستان دو تا سوال داشتم :19:

1- چرا منفی در منفی مثبت میشود؟

2- جذر چگونه بوجود آمد ؟
:41:
اگر کسی میدونه دریغ نکنه ممنون میشم :11:


ممنون از سوالای جالبتون خواهر(؟) ترانه!
در مورد سوال اولتون عرض کنم که:
چون دو طرف یه تساوی باید هم علامت باشن.اینطوری میشه:
ببینید معادله "منفی بی در منفی آ=منفی سی"رو با در نظر گرفتن a,b,c با علامت مثبت،در نظر بگیرید.
می تونیم تبدیشل کنیم به"منفی یک در بی در منفی یک در سی=منفی یک در آ"حالا شما از هر طرف این تساوی یک منفی حذف کنید.اینجاست که به تناقض می خوریم.ویک عدد مثبت یعنی سی با یک عدد منفی یعنی منفی آبی برابر میشه.(البته این روش به شرطی درسته که قبول کنیم مثبت در مثبت میشه مثبت.امیدوارم شما قبول داشته باشید!)
برای سوال دوم باید بگم:
احتیاج بشر باعث کشف جذر شد!مثلا معادله x+1=0 منجر به معرفی اعداد منفی شد.x^2-4=0 (مثلا) هم موجب شد تا یه چیزی به اسم ریشه دوم اختراع بشه(برای یافتنx) یا مثلا وقتی دلتای معادله ی درجه دو منفی شد اعداد موهومی و از اونجا مختلط بوجود اومدن.
بازم سوالی بود در خدمتم.
با تشکر.
خواهشا گوش کنین!. ما چیزی به نام منفی در منفی میشه مثبت نداریم.!و از دو طرف یک تساوی منفی ها رو ببریمم نداریم.!
برای حل این سوال شما باید اصولی به نام اصول میدان رو بدونی دو تا از بند ها ی این اصول اینه که:
-مجموعه اعداد حقیقی نسبت به جمع دارای عضو خنثی است
-هر عضو مجموعه اعداد حقیقی دارای عضو قرینه است
منظور عبارت اول اینه که R یک عضو داره که در جمع با هر عدد حقیقی خود عدد حقیقی به دست میاید (که در جمع و تفریق امروزی 0 عضو خنثی است)
برای عبارت دوم فک کنم فقط تعریف عضو قرینه رو باید بگم عضو قرینه یک عضو ,عضوی است که با جمع با آن عضو ,عضو خنثی حاصل شود(چی شد!:31:)(که در جمع وتفریق امروزی مثلا قرینه 2 میشه2- که داریم0= 2+(2-))
عضو قرینه a رو باa- نشان میدهند
حالا به کمک این اصول(منظورم فقط این 2 تا نیستا) قضایی زیر اثبات میشه

[ برای مشاهده لینک ، لطفا با نام کاربری خود وارد شوید یا ثبت نام کنید ]



.....

من اولیشو ثابت میکنم:





[ برای مشاهده لینک ، لطفا با نام کاربری خود وارد شوید یا ثبت نام کنید ]


توضیح:به جای a- 'گذاشتیم x بعد به این نتیجه رسیدیم a قرینه ی x است و به عبارت سوم رسیدیم و دوباره به جای x 'گذاشتیم a- و به حکم رسیدیم بازی قشنگی بود نه! ولی ریاضیات یعنی اثبات منطقی
ضمیمه:



1.از اینکه گفتم جمع وتفریق امروزی برای این بود که اگه ما تابع جمعو تریفشو تغییر بدیم مثلا تابعی به شکل

[ برای مشاهده لینک ، لطفا با نام کاربری خود وارد شوید یا ثبت نام کنید ]

تعریف بشه اون وقت عضو خنثی به جای 0 میشه 1- ! و مثلا قرینه 2 به جایه 2- میشه 4- ! جالبه نه!!!.

2.توجه کنین منفی تفریق با منفی قرینه فرق میکنه در واقع اولی عمل دوتایی و دومی عمل یکتایی است
بزارین این طوری توضیح بدم
تعریف تابع تفاضل:



[ برای مشاهده لینک ، لطفا با نام کاربری خود وارد شوید یا ثبت نام کنید ]

تعریف تابع قرینه:

[ برای مشاهده لینک ، لطفا با نام کاربری خود وارد شوید یا ثبت نام کنید ]

در واقع تفاضل دو عدد xوy را میگیره و x را با قرینه y یعنی (y-) جمع میکنه وما اینکارو با x-y نشون میده(عمل دوتایی)
ولی تابع قرینه عضو X را میگیره و x- را که قرینه ی x است تحویل میده(عمل یکتایی).!
ریاضی خیل با حاله ! اول به ما میگه 1+1=2 بعد میگه منفی درمنفی مثبت بعد دو طرف تساوی یه عدد بود مبریم! صورت مخرج و میبریم ساده میکنیم!و در آخر وقتی به سن بلوغ ریاضی رسیدی !!!.میفهمی همه این قضایی بوده که اثبات منطقی داشته و ثابت شده و فقط برای اینکه ما بهمیم اونطوری به ما گفتن.در واقع ما ثابت میکنیم 1+1=2درسته.!
درباره تاریخچه ی جذر هم که بدونی کی ایجاد کرده و اینا! تاریخچه ریاضی کاره maxwell-1989 جان است ولی اگه انگلیسیت خوبه یه سرچ تو نت یا اینجا شاید کمکت کنه

برای مشاهده محتوا ، لطفا وارد شوید یا ثبت نام کنید

chessmathter
29-09-2008, 22:22
آهان یادم رفت! بعدشم جایه این سوالات تو یه اتاق ریاضیات (طرح سوالات )است.:5::46:

برای مشاهده محتوا ، لطفا وارد شوید یا ثبت نام کنید

tarane
29-09-2008, 23:41
سلاممم دوستان !:11::11::11::11::11::11::11::11::11::11::11::11:
ممنون یه هفته بود داشتم دور خودم میگشتم و جوابی پیدا نمیکردممم:10: (با اینکه حالا هم نفهمیدمم چی شد:37::45:) باز کارم راه افتادددد خیلی متشکرمممممم :40:خیلی ممنوننننن دستتتون درد نکنه:38: !در مورد جذر هم من خیلی بیسوادممم:23: زبان انگلیسیمم:9: ....:42: :19:

chessmathter
30-09-2008, 00:20
سلاممم دوستان !:11::11::11::11::11::11::11::11::11::11::11::11:
ممنون یه هفته بود داشتم دور خودم میگشتم و جوابی پیدا نمیکردممم:10: (با اینکه حالا هم نفهمیدمم چی شد:37::45:) باز کارم راه افتادددد خیلی متشکرمممممم :40:خیلی ممنوننننن دستتتون درد نکنه:38: !در مورد جذر هم من خیلی بیسوادممم:23: زبان انگلیسیمم:9: ....:42: :19:

آدم دلش ریش ریش میشه:31:! از من به تو نصیحت تو زندگی خودتو دست کم نگیر.:46::5:
آقا بذار یه طور دیگه بگم :ما میگیم منفی در منفی میشه مثبت چون قرینه ی قرینه یه عدد میشه خودش!

a)=a-)-

ولی ما تو محاسباته روزمره همش که نمیتونیم بگیم: آهان حالا قرینه قرینه این شده خودش اینجا حالا باید اثبات کنیم

( ab=(-a)*(-b !
واسه اینکه ملت گیج نشه و بچه ها راحت موضوع رو بگیرن به عمد و غلط مگیم! منفی در منفی میشه مثبت.!
(البته کسی که بعد بره ریاضی محض اساتید از اشتباه درش میارن.!:31:)
در واقع اثبات منفی در منفی میشه مثبت یعنی اثبات قرینه ی قرینه یه عدد میشه خودش.!
درباره جذرم اومیدوارم maxwell_1989 یه تاریخچه خوب پیدا کنه.
اینم یه فارسیش ولی فکر نکنم کمکت کنه

برای مشاهده محتوا ، لطفا وارد شوید یا ثبت نام کنید

m1367m2006
30-09-2008, 12:09
چرا جذر وجود داره و اعداد منفی جذر ندارند توصیه می کنم یه کتاب انالیز حقیقی مثه اصول انالیز ریاضی
یه قضیه توپ با اثباتش توشه!

chessmathter
30-09-2008, 14:54
چرا جذر وجود داره و اعداد منفی جذر ندارند توصیه می کنم یه کتاب انالیز برداری مثه اصول انالیز ریاضی
یه قضیه توپ با اثباتش توشه!
کی گفته عدد منفی جذر نداره اینجاست که عدد مختلط i میاد وسط:31:.!

[ برای مشاهده لینک ، لطفا با نام کاربری خود وارد شوید یا ثبت نام کنید ]

eshghe eskate
30-09-2008, 15:12
چرا جذر وجود داره و اعداد منفی جذر ندارند توصیه می کنم یه کتاب انالیز برداری مثه اصول انالیز ریاضی
یه قضیه توپ با اثباتش توشه!
ولی جدی چرا اعداد منفی چذر ندارند؟؟؟؟؟؟چون توان دو هیچ عددی منفی نیس؟؟
اصلا از کجا به دست اومده که قرینه ی یه عدد میشه منفیه اون عدد؟؟اینو میشه اثبات کرد؟

chessmathter
30-09-2008, 15:31
ولی جدی چرا اعداد منفی چذر ندارند؟؟؟؟؟؟
اصلا از کجا به دست اومده که قرینه ی یه عدد میشه منفیه اون عدد؟؟اینو میشه اثبات کرد؟
انقدر خودمونو کشتیم باز حرف خودتونو میزنین.!!! دی...
بابا 3 تا پست بالاتر و بخونید
درمورد جذر اعداد منفی این اولر جون کند تا بگه عدد منفی جذر داره یه نگه اینجا بندازین

برای مشاهده محتوا ، لطفا وارد شوید یا ثبت نام کنید
در مورد سواله دوم نه نمیشه.
این پست و میشه با دقت بخونین

برای مشاهده محتوا ، لطفا وارد شوید یا ثبت نام کنید
تعریفه اون علامت (منفی) برای نشون دادن عضو قرینه a است
سوالت مثل این میمونه که بگیم چرا دو رو مینویسیم 2.!

m1367m2006
30-09-2008, 15:52
chessmather جان منظورم اعداد حقیقیه نه مختلط!

اصلا از کجا به دست اومده که قرینه ی یه عدد میشه منفیه اون عدد؟؟اینو میشه اثبات کرد؟یعنی چی? این یه تعریفه! شما یه چیز دیگه تعریف کن مهم اینکه ازش کجا استفاده بشه این فقط یه نماده!

Maxwell_1989
30-09-2008, 20:11
اصلا از کجا به دست اومده که قرینه ی یه عدد میشه منفیه اون عدد؟؟اینو میشه اثبات کرد؟
این تعریفه.میلاد جون نمادگذاری هم یه بخشی از تعریفه مگه نه؟

m1367m2006
30-09-2008, 21:43
بله اصل خصوصیاته شما دو را فارسی . لاتین و............ مینویسی فرق داره؟

mahdi.a81
09-10-2008, 23:57
5 تا آدم دور یک میز گرد به چند حالت میتونن بشینن

سلام دوست عزير :
5 تا آدم
خوب نفر اول مهم نيست كجا بشينه جون ميز گرده و اول وآخر نداره پس هرجا بشينه يه حالت حساب ميشه
نفر دوم تو 4 موقعيت مونده حق انتخاب داره .حالا جاي نشستن فرق داره
نفر سوم 3 حالت داره ، نفر چهارم 2 حالت و نفر پنجم 1 حالت كه اگه در هم ضرب كني ميشه !4
به طوز كلي براي n نفر !(n-1) راه وجود داره

mahdi.a81
10-10-2008, 15:19
تعداد حالت هایی که میشه 8 تا وزیر تو خونه های شطرنج گذاشت که هیچ کدوم همدیگرو تهدید نکنن چند تاست؟
من خودم بعد از یه ایده سوال و به این convert !کردم
تعداد توابع از {1,2,3,4,5,6,7,8} به {1,2,3,4,5,6,7,8} که یک به یک و پوشاست و جمع مولفه اول و دوم و تفاضل مولفه دوم از مولفه اول هر زوج مرتبش متفاوت باشه؟
مثلا این یکیشه
{(1,4),(2,6),(3,1),(4,5),(5,2),(6,8),(7,3),(8,7)}
{15,10,14,7,9,4,8,5} جمع مولفه های هر زوج مرتب
{2,4,3-,3,1-,4,2-,1-} تفاضل مولفه دوم از مولفه اول
فعلا اینجا هنگ کردم :31::13:

اگه کسی تونست بقیه ایده منو بره و حل کنه بگه اگه هم راه حل سوال و میدونه یا حل کرده بگه(سوال اصلی همون صفحه شطرنج ست)


سلام دوست عزيز
من راه حل كاملش يادم نيست ولي ميدونم توي مسائل درس طراحي الگوريتم از روش بازگشت به عقب قابل حله . اگه راهش رو ديدم توضيح ميدم .
البته بگم توي درس طراحي الگوريتم فقط الگوريتم رو مينويسند و نه جوابهايي كه از حلش بدست مياد
موفق باشيد .

caca_caca888
10-10-2008, 18:30
پس منم یه چی بگم...

شما به این جمله توجه کنید:
نمیخواهم نروم...
- در -
یعنی چی؟؟

یعنی میخواهم بروم...
+

حالا

نمیخواهم بروم...
- در +
یا میخواهم نروم...
+ در -
جفتشون یعنی نمیخام برم...
-

حالا میخواهم بروم...
+ در += +

پس در نتیجه میخواهم بروم = نمیخواهم نروم...
- در - = + در +

خیلی مثال ساده و قشنگیه...

mahdi.a81
11-10-2008, 00:21
سلام دوستان منم يه توضيح بدم با دادن يه منبع
ببينيد ما يه چيزايي توي رياضيات داريم كه فك كنم بهش ميگن اصول اوليه يا تعاريف
مثل اينكه ..........
الان يادم نيست
حالا وقتي ميهوايم تو يه فضا بحث كنيم (مثل همين فضايي كه داريم و اسمشم يادم نيست هيچ كسي هم نميدونه كه اين چيه) يه تعاريف داريم . تو كتاب جبر خطي بجه هاي رياضي (كه هم محض ها ميخونن هم كاربرديها ) 11 -12 تا اصل نوشته كه دوستمون chessmather جند تاشو بيان كرد
كه البته اثبات هم داره .
در مورد سوال اول توضيحات دوستان خيلي عالي بود مخصوصا توضيحادبي كاكاي عزيز . شما خودت هم فك كني چسزاي جديد بدست مياري
در مورد سوال دوم به گفته يكي از اساتيد انسانها اول تنها اعداد طبيعي رو ميشناختن ؛ يعني چي ، يعني براي شمارشها از اعداد طبيعي ( نمادهاي اونها) استفاده ميكردند . بعدش نياز به وجود اعداد منفي و صفر احساس شد . با وجود آمدن اين اعداد ، كم كم نياز انسان به اعداد گويا ، اين اعداد رو مطرح كرد( مثلا تقسيم يه سيب به 4 قسمت) . بعدش اعداد اصم كه البته بعضي از جذرها هم در اين بازه قرار ميگيرند . مثلا همه ميدونند كه انسانها وقتي در حل معادله معروف x^2 = 2 به جايي نرسيدند نماد "من در آوردي " جذر بوجود آمد .
يادمون باشه تا در مكاني نيازي نباشه به وجود چيزي اون چيز بوجود نمياد . پس هيچ چيزي از روي بيكاري مطرح نميشه و ريشه علمي داره .توي رياضيات اين نظم بسيار چشم نوز تره
البته اين يه نتيجه گيري بود نه خداي نكرده يه توهين به دوست خوبمون .
ببخشيد تبديل به يه سخنداني شد .
خبر خطي و جبر بحث سوال يك توشون هست .
البته جبر دانشگاه
موفق باشيد .

m1367m2006
11-10-2008, 08:27
سلام دوستان منم يه توضيح بدم با دادن يه منبع
ببينيد ما يه چيزايي توي رياضيات داريم كه فك كنم بهش ميگن اصول اوليه يا تعاريف
مثل اينكه ..........
الان يادم نيست
حالا وقتي ميهوايم تو يه فضا بحث كنيم (مثل همين فضايي كه داريم و اسمشم يادم نيست هيچ كسي هم نميدونه كه اين چيه) يه تعاريف داريم . تو كتاب جبر خطي بجه هاي رياضي (كه هم محض ها ميخونن هم كاربرديها ) 11 -12 تا اصل نوشته كه دوستمون chessmather جند تاشو بيان كرد
كه البته اثبات هم داره .
در مورد سوال اول توضيحات دوستان خيلي عالي بود مخصوصا توضيحادبي كاكاي عزيز . شما خودت هم فك كني چسزاي جديد بدست مياري
در مورد سوال دوم به گفته يكي از اساتيد انسانها اول تنها اعداد طبيعي رو ميشناختن ؛ يعني چي ، يعني براي شمارشها از اعداد طبيعي ( نمادهاي اونها) استفاده ميكردند . بعدش نياز به وجود اعداد منفي و صفر احساس شد . با وجود آمدن اين اعداد ، كم كم نياز انسان به اعداد گويا ، اين اعداد رو مطرح كرد( مثلا تقسيم يه سيب به 4 قسمت) . بعدش اعداد اصم كه البته بعضي از جذرها هم در اين بازه قرار ميگيرند . مثلا همه ميدونند كه انسانها وقتي در حل معادله معروف x^2 = 2 به جايي نرسيدند نماد "من در آوردي " جذر بوجود آمد .
يادمون باشه تا در مكاني نيازي نباشه به وجود چيزي اون چيز بوجود نمياد . پس هيچ چيزي از روي بيكاري مطرح نميشه و ريشه علمي داره .توي رياضيات اين نظم بسيار چشم نوز تره
البته اين يه نتيجه گيري بود نه خداي نكرده يه توهين به دوست خوبمون .
ببخشيد تبديل به يه سخنداني شد .
خبر خطي و جبر بحث سوال يك توشون هست .
البته جبر دانشگاه
موفق باشيد .
اولا من همین جا گفتم که جذر من درآوردی نیست یعنی اینکه ریشه n ام هر عدد قابل اثباته!
دوما شما چی میخونید
اخه پروفایلتون بستس؟
سوما شما هنوز یه ماه نیست عضو اینجایی 100 پست هم که بدی هنوز عضو جدیدی

در مورد سوال دوم به گفته يكي از اساتيد انسانها اول تنها اعداد طبيعي رو ميشناختن ؛ يعني چي ، يعني براي شمارشها از اعداد طبيعي ( نمادهاي اونها) استفاده ميكردند . بعدش نياز به وجود اعداد منفي و صفر احساس شد . با وجود آمدن اين اعداد ، كم كم نياز انسان به اعداد گويا ، اين اعداد رو مطرح كرد( مثلا تقسيم يه سيب به 4 قسمت) . بعدش اعداد اصم كه البته بعضي از جذرها هم در اين بازه قرار ميگيرندتایید میشه

بهش ميگن اصول اوليه يا تعاريفمفاهیم اولیه مثه نقطه
رابطه های اولیه مثه خط
اصول موضوعه مثه وجود داشتن مجموعه
حالا منظورت کدومه؟:31:

chessmathter
11-10-2008, 14:49
سلام دوستان منم يه توضيح بدم با دادن يه منبع
ببينيد ما يه چيزايي توي رياضيات داريم كه فك كنم بهش ميگن اصول اوليه يا تعاريف
مثل اينكه ..........
الان يادم نيست
حالا وقتي ميهوايم تو يه فضا بحث كنيم (مثل همين فضايي كه داريم و اسمشم يادم نيست هيچ كسي هم نميدونه كه اين چيه) يه تعاريف داريم . تو كتاب جبر خطي بجه هاي رياضي (كه هم محض ها ميخونن هم كاربرديها ) 11 -12 تا اصل نوشته كه دوستمون chessmather جند تاشو بيان كرد
كه البته اثبات هم داره .
در مورد سوال اول توضيحات دوستان خيلي عالي بود مخصوصا توضيحادبي كاكاي عزيز . شما خودت هم فك كني چسزاي جديد بدست مياري
در مورد سوال دوم به گفته يكي از اساتيد انسانها اول تنها اعداد طبيعي رو ميشناختن ؛ يعني چي ، يعني براي شمارشها از اعداد طبيعي ( نمادهاي اونها) استفاده ميكردند . بعدش نياز به وجود اعداد منفي و صفر احساس شد . با وجود آمدن اين اعداد ، كم كم نياز انسان به اعداد گويا ، اين اعداد رو مطرح كرد( مثلا تقسيم يه سيب به 4 قسمت) . بعدش اعداد اصم كه البته بعضي از جذرها هم در اين بازه قرار ميگيرند . مثلا همه ميدونند كه انسانها وقتي در حل معادله معروف x^2 = 2 به جايي نرسيدند نماد "من در آوردي " جذر بوجود آمد .
يادمون باشه تا در مكاني نيازي نباشه به وجود چيزي اون چيز بوجود نمياد . پس هيچ چيزي از روي بيكاري مطرح نميشه و ريشه علمي داره .توي رياضيات اين نظم بسيار چشم نوز تره
البته اين يه نتيجه گيري بود نه خداي نكرده يه توهين به دوست خوبمون .
ببخشيد تبديل به يه سخنداني شد .
خبر خطي و جبر بحث سوال يك توشون هست .
البته جبر دانشگاه
موفق باشيد .
اصل اثبات نداره اون قضیه است که اثبات داره.اصل پذیرفته شده است !!!!:31::5:

chessmathter
11-10-2008, 14:55
سلام دوست عزيز
من راه حل كاملش يادم نيست ولي ميدونم توي مسائل درس طراحي الگوريتم از روش بازگشت به عقب قابل حله . اگه راهش رو ديدم توضيح ميدم .
البته بگم توي درس طراحي الگوريتم فقط الگوريتم رو مينويسند و نه جوابهايي كه از حلش بدست مياد
موفق باشيد .
من الگوریتمی که از یه جواب به جواب های دیگه برسه یا جوابارو بدست بیاره یا جوابا رو نمیخوام فقط تعداد جواب ها. ها.!!!

mahdi.a81
12-10-2008, 09:58
صفحه مختصات x,y را در نظر بگیرید.
یک جسم روی نقطه 0,0 قرار دارد.می خواهیم این جسم را به نقطه (m,n) منتقل کنیم.
بدین منظور حرکت روی صفحه از یکی از معادلات زیر تبعیت می کند:

x,y>>>>x+1,y
or
x,y>>>>x,y+1
or
x,y>>>>x+1,y+1

یعنی هر حرکت یا به سمت راست است یا بالا و یا به صورت قطری به شمال شرق !
به چند طریق می توان این جسم را به نقطه (m,n) برد ؟



سلام
يكي از دوستان راه حل رو گفتند
من تكميل ميكنم

T(m,n) =T(m,n-1)+T(m-1,n)+T(m-1,n-1)+y
y=0
T(1,0) = 1
T(0,1) = 1
T(0,0) = 0


البته حل اين معادله بازگشتي كار ساده اي نيست و البته خيلي هم قشنگه اگه حل كردم ميذارم
علتش هم اينه كه دو تا مجهول( عامل بازگشتي) داره

Parser
16-10-2008, 07:51
من الگوریتمی که از یه جواب به جواب های دیگه برسه یا جوابارو بدست بیاره یا جوابا رو نمیخوام فقط تعداد جواب ها. ها.!!!

سلام
جواب ميشه هشت فاكتوريل به توان دو

mahdi.a81
18-10-2008, 23:31
سلام
جواب ميشه هشت فاكتوريل به توان دو


سلام
اگه توضيح بدي ممنون ميشيم
البته من راه حل اصلي رو ديدم
اما الان وقت گشتن رو ندارم
راه حل قشنگي داره

chessmathter
19-10-2008, 16:45
سلام
جواب ميشه هشت فاكتوريل به توان دو


سلام
اگه توضيح بدي ممنون ميشيم
البته من راه حل اصلي رو ديدم
اما الان وقت گشتن رو ندارم
راه حل قشنگي داره
دقیقا میشه بگی این عدد از کجا آمد.؟!

vampire3250
20-10-2008, 16:17
سلام ...:31:
من یه سوال داشتم ، میخواستم جوابشو ببینم ...:19:

سوال :

K را چنان بیابید تا ریشه های معادله ی زیر تشکیل تصاعد عددی دهند :

[ برای مشاهده لینک ، لطفا با نام کاربری خود وارد شوید یا ثبت نام کنید ]


ممنون میشم اگه جوابشو بدین ...:10:

STON-COLD
22-10-2008, 21:23
میخواستم بدونم به توان منفی چطور حساب میشه
مثلا 2 به توان منفی2

Maxwell_1989
22-10-2008, 22:04
آ بتوان منفی ان=یک تقسیم بر(آ بتوان ان)
تشکر

chessmathter
23-10-2008, 00:17
میخواستم بدونم به توان منفی چطور حساب میشه
مثلا 2 به توان منفی2


آ بتوان منفی ان=یک تقسیم بر(آ بتوان ان)
تشکر
با تشکر از maxwell و برای فهم بیشتر.!!!

[ برای مشاهده لینک ، لطفا با نام کاربری خود وارد شوید یا ثبت نام کنید ]

bakbari
27-10-2008, 16:09
سلام . خوبید ؟ یه مسئله دارم .می خوام اثباتش کنم .اگه کسی میدونه برام اثباتش رو کامل به همراه چندتا مثال بزاره .ممنون . سوال : بدون عمل تقسیم ، چگونه می توان خارج قسمت (P(x بر ax+b را بدست آورد ؟

m1367m2006
27-10-2008, 16:29
اینا درست یادم نیست ولی تو کتاب حسابان فک کنم باید باشه
اگه نشد بگو خودم نگاه میکنم

bakbari
27-10-2008, 17:51
سلام . تو کتاب حسابان ندیدم . اگه میشه زودتر برام ردیفش کن . ممنون .

m1367m2006
27-10-2008, 20:56
برای بدست اوردن باقی مانده تقسیم چند جمله ای بر کافی است ریشه مقسوم علیه را بدست اورده و در مقسوم علیه به جای قرار دهیم
[ برای مشاهده لینک ، لطفا با نام کاربری خود وارد شوید یا ثبت نام کنید ]
[ برای مشاهده لینک ، لطفا با نام کاربری خود وارد شوید یا ثبت نام کنید ]
اقا شرمنده انگار شما خارج قسمت را می خواستید من سوال را بد خوندم حالا براب چی میخواید باید فک کنم

Maxwell_1989
28-10-2008, 09:03
توضیحش از این طریق سخته(یعنی از طریق کامپیوتر) چون جدول کشیدن و... میخواد.شما یه سرچی بکن(منتها بنویس هورنر) اگه چیزی دستگیرت نشد بگو یه کاریش می کنیم.

rezamahmoodi58
30-10-2008, 23:18
با سلام از دوستان خواهشمندیم روشی رو معرفی نمائید که بتوان طول یک لوله که داده های ما فقط به طور مثال تشکیل شده از چندین نقطه فضایی با شعاعهایی در انحنای بطور مثال

نقطه 1) x=215 y=-146 z= 462.5 شعاع r=0
نقطه 2) x=150 y=-146 z= 462.5 شعاع r=30
نقطه 3) x=150 y=-37.4 z= 462.5 شعاع r=30
نقطه 4) x=139.1 y=-29.5 z= 333.5 شعاع r=30
نقطه 5) x=135.5 y=189.1 z= 298.9 شعاع r=30
نقطه 6) x=76.8 y=189.1 z= 292.7 شعاع r=0

که طول این لوله 540.1 هستش حال آیا کسی میتونه روشی جهت محاسبه پیشنهاد کنه

Niels Henrik Abel
31-10-2008, 12:40
سلام در مورد این که منفی از کجا اومده و جذر چیه و .... اولا خوشحالم این سوالا بالاخره تو ذهن یکی اومده کاراییه که 100و150 سوال روش وقت گذاشتن رو بچه های ما تو 1 روز می خوان یاد بگیرن....من سال اول دانشگاهم رشته ریاضی محض و خیلی خیلی خوشحال شدم بابت سوالت
در مجموع از اقایی که درباره میدان و گروهها صحبت کردن متشکرم
این ایده رو اولین بار مصری ها دادن در واقع بعد از پدیدار شدن اعداد صحیح و معیاری برای اون( که بررسی ماهیت اون اینجا مهم نیست)تمام سوالات و معادلات در همین مجموعه جواب داشتن ولی روند اقتصادیه مصری ها طوری بود که یه سری کارا مثل تقسیم بندی زمین و غذا که قبلا براش معیار تعیین شده بود طوری پیش رفت که مجبور شدن واحد ها رو کوچکتر کنن ولی این کار بعد یه مدت احمقانه جلوه کرد و اومدن یه معیار واحد رو رد نظر گرفتن و قسمتهای کوچکتر را با الفاضی چون نصف و...بکار بردن واحد های منفی رو هم یونانی ها بعدا براساس این معیار اوردن تو کار این در مورد اعداد بود و جذر هم حل یه معادله محض در یونان بود زیاد روی کلمه ها و ...تکیه نکن اگه روند تدریجی پیدایش اعداد و ... در ریاضی پیش بگیری مشکلت حله و به مفاهیم ارزنده تری میرسی تو یه مجموعه یا setهم ما یه مبنا و عضو خنثی داریم داریم و الفاظ قرینه و ... تعریف می شن ببین باید به الفاظ تعریف اصول و... مسلط باشی ولی همون طور که دوستمون کفت یه کتاب انالیز بخونی حله

sunrise
01-11-2008, 00:59
با سلام،

ابتدا اين مطلب رو خاطرنشان ميشم که من تابلوي اعلانات که راجع به نحوه استفاده از انجمن رياضيات بود رو مطالعه کردم و تاپيک هايي که در اين مورد وجود داره رو نيز بررسي کردم منتها تاپيک مخصوصي براي رياضيات گسسته نديدم و با توجه به اينکه سؤالات من در اين مورد هست فکر کردم اگه يه تاپيک با اين نام در انجمن وجود داشته باشه بهتر باشه اما اگه مديران محترم انجمن رياضيات احساس مي کنند که اين تاپيک بايد با تاپيک ديگه اي ادغام بشه صاحب اختيار هستند :46:

خب بريم سر سؤالات، البته اين رو هم بگم که من جواب اين سؤالات رو تا فردا (حداکثر ساعت 10:30 صبح) ميخوام.

1) روابط زير را اثبات کنيد، اگر P و Q و S گزاره باشند.

[ برای مشاهده لینک ، لطفا با نام کاربری خود وارد شوید یا ثبت نام کنید ]

2) آيا استنتاج هاي زير معتبر است؟

[ برای مشاهده لینک ، لطفا با نام کاربری خود وارد شوید یا ثبت نام کنید ]

با تشکر

elendil
04-11-2008, 19:40
دوستان میشه این رو که اگه توی یک جایگشت دایره ای یک نفرو ثابت بگیریم بقیش میشه همون جایگشت خطی رو اثبات کنید ؟ من سوم دبیرستانم

m1367m2006
04-11-2008, 19:57
یعنی چی
خوب بببین n تا مکان و شی داریم یه مکان را با یه شی پر میکنیم برای بقیه طبق قاعده ضرب که با هاش n جایگشت را اثبات میکنیم داریم n-1 فاکتوریل

elendil
04-11-2008, 21:31
حاجی جان این توجیهش هست نه اثبات

m1367m2006
05-11-2008, 11:35
حاجی جان این توجیهش هست نه اثبات
شما اصلن جایگشت n گردایه را بلدی؟

atre_hayer60
05-11-2008, 21:52
سلام بچه ها!

فقط تو رو خدا کمک کنید و این مساله رو حل کنید چون استادمون گفته اگه حل نکنید حذفش کنید!!!!!

اعضاء یک باشگاه ورزشی 50 نفر می باشند. 20 نفر عضو تیم فوتبال، 20نفر عضو تیم والیبال، 15 نفر هم عضو تیم بسکتبال هستند. 3 نفر عضو مشترک والیبال و بسکتبال هستند و 6 نفر عضو مشترک فوتبال و والیبال و 5 نفر نیز عضو مشترک فوتبال و بسکتبال می باشند و 3 نفر هم عضو هیچ تیمی نیستند!
الف) مطلوب است تعداد افرادی که عضو هر سه تیم هستند؟
ب) مطلوب است تعداد افرادی که عضو تیم فوتبال هستند؟
ج) مطلوب است تعداد افرادی که فقط عضو تیم والیبال هستند؟
د) مطلوب است تعداد افرادی که فقط عضو تیم بسکتبال هستند؟


بچه ها تو رو خدا کمک کنید که کارم گیره... فقط تا فردا وقت هست!

مرسی

M . Sayid
05-11-2008, 23:02
این که احتمال 90 درصد همون محاسبه پی A و پی B و پی آ پریم جبر و احتمال سال سوم باشه ( ریاضی-فیزیک) چون یادمه یه اینجور چیزایی بود همین قدر یادمه شرمنده فکر می کنم بهش می گفتن احتمال هم شانس

Parser
06-11-2008, 06:42
سلام

فرض كنيد يك صفحه ي شطرنج 8*8 داريد، براي قرار گرفتن يك مهره ي رخ روي آن 64 حالت وجود دارد.
اكنون تصور كنيد كه آنرا روي يكي از آن خانه ها گذاشتيم، رخ بعدي روي خانه هايي به جز خانه ي انتخاب شده ي قبلي و سطر و ستون متناظر با آن قرار مي گيرد، پس 49 حالت براي انتخاب وجود دارد.
( براي اينكه بهتر متوجه بشويد، تصور كنيد كه مهره ي اول روي خانه اي در گوشه ي صفحه قرار مي گيرد. اگر آن سطر و ستون را حذف نماييم، يك صفحه ي 7*7، يعني 49 خانه براي انتخاب باقي مي ماند.)
همچنين با گذاشتن مهره ي دوم 36 خانه براي انتخاب مكان مهره ي سوم باقي مي ماند.
به همين ترتيب در نهايت براي مهره ي هشتم فقط يك خانه باقي مي ماند.
طبق اصل ضرب همه ي اين اتفاقات در 64*49*36*25*16*9*4*1 طريق مي تواند اتفاق بيفتد.
يا به عبارتي در هشت فاكتوريل به توان دو صورت.

m1367m2006
06-11-2008, 08:57
داداش از اصل شمول طرد استفاده کن

nemesis
08-11-2008, 11:43
اگه امکان داره این معدله رو برام حل کنید !
لطفاً قدم به قدم باشه !



5x+9y+1-x-y=10x+3y+8-8x-8y=-x+5y+9-9x-9y

msnasiri
08-11-2008, 15:35
5x+9y+1-x-y=10x+3y+8-8x-8y=-x+5y+9-9x-9y

4x+8y+1=2x+8-5y=9-10x-4y

4x+8y+1=2x+8-5y
2x+8-5y=9-10x-4y


2x+13y=7
12x-y=1


-12x-78y=-42
12x-y=1
=======
-79y=-41
y=79/41
and x=....

البته ممکنه جایشو اشتباه کرده باشم